53
8/9/2019 Behavioral Science Questions http://slidepdf.com/reader/full/behavioral-science-questions 1/53 BEHAVIORAL SCIENCES Two young boys are playing at a daycare center. One holds a ball on top of some blocks that the other child has  placed on the floor. The second child helps steady the blocks, then the first child lets go of the ball, knocking the  blocks down to the floor. They both watch and then repeat the process. These children are most likely  A. 10 months old  B. 16 months old  . 1! months old  ". #$ months old  %. $! months old  %&planation'  The correct answer is %. The capacity for cooperati(e play generally does not begin much before the age of $.  )rior to this time *#$+0 months-, children may play in a parallel fashion, but without real interaction. A #+year+old woman (isits a primary care physician complaining of recurrent nausea and generalied abdominal  pain, which has distressed her and compromised her functioning for the past se(eral months. Thorough  e(aluations by a gynecologist and gastroenterologist ha(e re(ealed no abnormalities. /he was referred to a  neurologist for headaches and has been taking acetaminophen for her oint pains. The primary care doctor  completes a thorough physical e&am2 the results are normal. 3hich of the following is the most likely diagnosis4  A. Body dysmorphic disorder  B. on(ersion disorder  . 5actitious disorder  ". alingering  %. /omatiation disorder  %&planation'  The correct answer is %. )atients with somatiation disorder ha(e many medically une&plained symptoms in  multiple body systems, causing work limitation, increased (isits to the physician, needless surgery, or  unnecessary medical treatments. /omatiation disorder is distinguished by its ego dystonic symptoms2 that is,  the patient7s functioning is compromised because of the unpleasantness of symptoms. 8t usually begins before  age 0. Body dysmorphic disorder *choice A- refers to the patient who is preoccupied with the belief that some part of  the body is marred in looks. 8t usually begins in adolescence and is e9ually common in males and females. on(ersion disorder *choice B- refers to patients with neurological complaints that are not consistent with  present+day knowledge about the ner(ous system *e.g., anesthesia that does not run along a ner(e 1

Behavioral Science Questions

Embed Size (px)

Citation preview

Page 1: Behavioral Science Questions

8/9/2019 Behavioral Science Questions

http://slidepdf.com/reader/full/behavioral-science-questions 1/53

BEHAVIORAL SCIENCES

Two young boys are playing at a daycare center. One holds a ball on top of some blocks that the other child has  placed on the floor. The second child helps steady the blocks, then the first child lets go of the ball, knocking the

  blocks down to the floor. They both watch and then repeat the process. These children are most likely

  A. 10 months old

  B. 16 months old

  . 1! months old

  ". #$ months old

  %. $! months old

  %&planation'

  The correct answer is %. The capacity for cooperati(e play generally does not begin much before the age of $.

  )rior to this time *#$+0 months-, children may play in a parallel fashion, but without real interaction.

A #+year+old woman (isits a primary care physician complaining of recurrent nausea and generalied abdominal

  pain, which has distressed her and compromised her functioning for the past se(eral months. Thorough

  e(aluations by a gynecologist and gastroenterologist ha(e re(ealed no abnormalities. /he was referred to a  neurologist for headaches and has been taking acetaminophen for her oint pains. The primary care doctor 

  completes a thorough physical e&am2 the results are normal. 3hich of the following is the most likely diagnosis4

  A. Body dysmorphic disorder 

  B. on(ersion disorder 

  . 5actitious disorder 

  ". alingering

  %. /omatiation disorder 

  %&planation'

  The correct answer is %. )atients with somatiation disorder ha(e many medically une&plained symptoms in

  multiple body systems, causing work limitation, increased (isits to the physician, needless surgery, or   unnecessary medical treatments. /omatiation disorder is distinguished by its ego dystonic symptoms2 that is,  the patient7s functioning is compromised because of the unpleasantness of symptoms. 8t usually begins before

  age 0.

Body dysmorphic disorder *choice A- refers to the patient who is preoccupied with the belief that some part of   the body is marred in looks. 8t usually begins in adolescence and is e9ually common in males and females.

on(ersion disorder *choice B- refers to patients with neurological complaints that are not consistent with  present+day knowledge about the ner(ous system *e.g., anesthesia that does not run along a ner(e

1

Page 2: Behavioral Science Questions

8/9/2019 Behavioral Science Questions

http://slidepdf.com/reader/full/behavioral-science-questions 2/53

  distribution-. A classic clue to this diagnosis is that the patient re(eals a relati(e lack of concern about the

  symptoms, known as la belle indifference.

There are three types of factitious disorder *5"2 choice -' 5" with psychological symptoms, 5" with medical  symptoms, and chronic 5". 5" with medical symptoms is different from somatiation disorder because in 5"

  symptoms are completely fabricated, patients often insist on hospitaliation and submit to in(asi(e procedures,

  and may produce symptoms through specific acts *e.g., taking drugs-. These patients differ from somatiation

  disorder in that they are consciously trying to assume a sick role.

alingering *choice "- refers to the situation in which a patient reports psychological or general medical

  symptoms in order to achie(e some easily recogniable secondary gain. The 9uestion implied no secondary

  gain for the patient, therefore malingering is not the best diagnosis in this case.

An !!+year+old male complaining of abdominal pain enters the emergency room with his wife. A mini+mental

  status e&am re(eals pronounced forgetfulness and confusion. The patient is disco(ered to ha(e acute

  appendicitis re9uiring immediate surgery. :e is unable to understand the situation and cannot pro(ide informed

  consent. 3hich of the following further actions must the physician take4

  A. "o not perform surgery

  B. :a(e another doctor confirm the necessity of surgery

  . Obtain a court order to perform surgery

  ". Obtain consent from his wife to perform surgery

  %. Try to persuade the patient to consent to surgery

  %&planation'

  The correct answer is ". 8n cases in which an emergency e&ists, the patient is incompetent to gi(e consent, and  the withholding of treatment would be potentially life+threatening, the physician must seek out close relati(es of 

  the patient to supply consent. The physician should proceed with treatment, assuming the patient would want

  the treatment had he or she understood the situation.

 ;ot performing surgery *choice A- could cost the patient7s life.

:a(ing another doctor confirm the necessity of surgery *choice B- is fa(orable *if done immediately- but not

  mandatory and does not change the patient7s consent status.

Obtaining a court order *choice - is not necessary with the patient7s wife immediately accessible.

Trying to persuade the patient to consent to surgery *choice %- would not only waste time and pro(e futile but  might agitate the patient as well.

A <0+year+old physician is reco(ering from aseptic meningitis that began two weeks ago. :e appears to ha(e lost

  considerable cogniti(e function, and he says he will not go on li(ing if his cogniti(e ability remains compromised.

  To demonstrate to him that reco(ery is occurring, serial e(aluations with which of the following psychological  assessments is indicated4

#

Page 3: Behavioral Science Questions

8/9/2019 Behavioral Science Questions

http://slidepdf.com/reader/full/behavioral-science-questions 3/53

  A. :alstead+=eitan Battery

  B. /tanford Binet 8ntelligence Test

  . >ineland Adapti(e Beha(ior /cale

  ". 3echsler Adult 8ntelligence /cale

  %. 3ide =ange Achie(ement Test

  %&planation'

  The correct answer is A. The :alstead+=eitan Battery is a group of tests that reflects the basic and higher le(el  cogniti(e and neuro+sensory functioning of the entire brain, and can be used in a serial fashion with little

  learning effect being present. /ince there is reco(ery of function for up to # years post ;/+trauma, test results

  can demonstrate that the present loss the patient is e&periencing is not permanent.

The /tanford Binet 8ntelligence Test *choice B-, used in the adult, mainly reflects (erbal skills and conse9uently  would miss large portions of this man7s situation.

The >ineland Adapti(e Beha(ior /cale *choice - assesses de(elopmental and social functioning, not cogniti(e  and neuro+sensory abilities.

  The 3echsler Adult 8ntelligence /cale *choice "- confines its results to intelligence assessment and does not

  assess more basic issues like aphasia and neuro+sensory skills.

The 3ide =ange Achie(ement Test *choice %- assesses academic achie(ement only.

?eraldine @ones, ."., is to see a #+year+old patient, @ohn /mith, whom she has ne(er talked with before. pon  entering the patient7s room, which of the following is the most appropriate introduction the physician can make4

  A. :ello, 87m "r. @ones.

  B. :ello, @ohn.

  . @ohn, 87m "r. @ones.

  ". r. /mith, 87m "r. @ones.

  %. r. /mith, 87m ?eraldine.

  %&planation'

  The correct answer is ". ommunication with patients should be le(eling. That is, if the physician e&pects to be

  addressed using a title, then the patient should also be addressed with a title.

  hoice A is not correct because the patient is not addressed by name2 and, the physician may be in the wrong

  room and about to treat the wrong person.

Page 4: Behavioral Science Questions

8/9/2019 Behavioral Science Questions

http://slidepdf.com/reader/full/behavioral-science-questions 4/53

  hoices B, , and % are not correct because there is no le(eling. 8n choice B, the physician is not identified. 8n

  choice , the patient is called by his first name while the physician maintains a distance through the title of "r.

  hoice % is incorrect because the patient is gi(en a title and the physician goes by her first name.

A 1+year+old female, who recently mo(ed from her family7s home in another state, is hospitalied for attempting

  suicide by taking an o(erdose of antidepressant medications. On the third day of her hospital stay, she insists,

  under threat of a lawsuit, that her medications be stopped and that she be discharged from the hospital so she

  can go home and finish the ob. :er sensorium is clear. :er physician should

  A. discharge her against medical ad(ice *AA-

  B. honor her re9uest and release her immediately

  . obtain an emergency order of detention

  ". release her to go back to her parents7 home

  %. sedate her 

  %&planation'

  The correct answer is . The physician should obtain an emergency order of detention, regardless of her   threats of a lawsuit. The woman clearly still has suicidal intent, demonstrated by her e&pressed (erbaliations,

  and is therefore a danger to herself.

hoices A, B, and " clearly place her in a position where she can carry out her plans to terminate her life.

  /edating her *choice %- is the second best choice since it will pre(ent her from taking her life2 howe(er,

  sedation does not gi(e therapists the opportunity to address the underlying moti(ations for her suicidal

  ideation.

%ight research scientists are brought into the hospital by the paramedics. They are suffering from diaphoresis,

  blurred (ision, palpitations, and hallucinations with brilliant colors. )olice suspect that the coffee at their lab

  meeting was laced with a psychoacti(e substance. 3hich of the following substances is most likely to be found in

  the coffee pot4

  A. Cysergic acid diethylamide *C/"-

  B. ethadone

  . )hencyclidine *))-

  ". )henobarbital

  %. Tetrahydrocannabinol *T:-

  %&planation'

  The correct answer is A. These eight researchers are probably under the influence of C/", which causes

$

Page 5: Behavioral Science Questions

8/9/2019 Behavioral Science Questions

http://slidepdf.com/reader/full/behavioral-science-questions 5/53

  hallucinations notable for their brilliant colors. C/" also shows acti(ity at serotonin receptors, and can acti(ate

  the sympathetic ner(ous system, resulting in symptoms such as diaphoresis, blurred (ision *due to pupil

  dilation-, and palpitations.

ethadone *choice B- is a synthetic opiate used to treat heroin addiction. 8t has analgesic properties, but does

  not ordinarily induce hallucinations.

)) *choice - can cause hallucinations marked by alterations of body image and distortions of space and time.  )) can also cause a dissociati(e anesthesia and analgesia. ommon side effects of )) use include

  hypersali(ation, muscular rigidity, hypertension, and nystagmus. :ighly colored (isual hallucinations are not as

  commonly seen with )) into&ication as with into&ication by C/".

)henobarbital *choice "- is a long+acting barbiturate that acts as a ;/ depressant. 8t is used in the long+term

  management of seiure disorders.

T: *choice %- is found in mariuana, and produces a euphoric high followed by subse9uent rela&ation and  sleepiness. ariuana use can result in (isual hallucinations, delusions, and a to&ic psychosis, but generally only

  at e&tremely high doses.

edical students at a maor teaching hospital are routinely assigned to obser(e obstetric patients and to assist

  during deli(ery. 3hen a male medical student introduces himself to an obstetric patient, the patient becomes  agitated and re9uests that no students be present during her deli(ery. The patient had been informed, prior to

  admission, that this was a teaching hospital and that a student would be assigned to her case. 3hen informed of 

  the patient7s refusal, the attending physician in charge should

  A. ask the patient7s husband for his consent

  B. assign a female medical student to obser(e

  . not allow any medical students to obser(e this patient

  ". ha(e the patient7s nurse seek permission

  %. ha(e the student approach the patient again and e&plain the necessity for student obser(ation

  5. ha(e the student obser(e in the background as a part of the health care team.

  ?. meet with the patient and discuss the (alue of obser(ation in medical training

  %&planation'

  The correct answer is . The patient has the right to decide who will or will not be present during her care. This  includes the right to refuse to be a part of a student7s educational e&perience. The desires of the patient, not  the physician or training facility, come first. 8f the patient does not want a medical student present during the

  deli(ery, respect the patient7s wishes.

hoice A is incorrect because the patient7s consent, not her husband7s, is re9uired. The husband cannot gi(e  consent for an alert, competent patient.

hoice B is incorrect because the patient is not reecting male students, but all students.

<

Page 6: Behavioral Science Questions

8/9/2019 Behavioral Science Questions

http://slidepdf.com/reader/full/behavioral-science-questions 6/53

  hoice " is incorrect because the patient has already refused. /ending the nurse to get permission suggests

  that the physician is not respecting her e&pressed wishes.

hoice % is incorrect because the patient has already refused. :a(ing the student go back and ask again will  only make the student uncomfortable and may make the patient angry.

  hoice 5 is incorrect because sneaking the student in to obser(e in the background is a direct contradiction of 

  the patient7s wishes.

hoice ? is incorrect because meeting with the patient to discuss the (alue of teaching encounters suggests

  putting pressure on the patient to change her mind. The purpose of the medical encounter is to seek the

  greatest benefit for the patient, not seek the best educational e&perience for the student. The patient7s wishes  predominate here.

A ##+year+old female college student is brought into the emergency room by the police, who found her walking

  back and forth across a busy street, talking to herself. The young woman appears to be oriented with respect to

  person, place, and time. :er first hospital admission was two months ago for a similar condition. "uring a

  psychiatric inter(iew, she has difficulty concentrating, and seems to hear (oices. A phone call to her sister 

  pro(ides the additional information that the girl dropped out of school three months ago and has been li(ing on  the street. rine to&icology is negati(e. This patient is most likely e&hibiting the signs and symptoms of

A. schioaffecti(e disorder 

  B. schioid personality disorder 

  . schiophrenia

  ". schiophreniform disorder 

  %. schiotypal personality disorder 

  %&planation'

  The correct answer is ". The patient is suffering from schiophreniform disorder. There has been a marked

  decline in the le(el of functioning and she was endangering herself in the middle of the street. /chiophreniform

  disorder is characteried by schiophrenia+like symptoms, but the duration of symptoms is less than si& months  *but more than one month-. 5ully de(eloped psychotic symptoms are typical.

8n schioaffecti(e disorder *choice A-, alterations in mood are present during a substantial portion of the illness.

  Although schioid personality disorder *choice B- produces detachment from social relationships and is

  characteried by restriction of emotional e&pression, it is not accompanied by a marked decline in occupational

  functioning.

8f the symptoms do not remit after si& months or more, then the diagnosis of schiophrenia *choice - should be

  made.

/chiotypal personality disorder *choice %- is characteried by eccentricities of beha(ior, odd beliefs or magical  thinking, and difficulties with social and interpersonal relationships. nlike schiophrenia, schiotypal personality

  disorder is not characteried by a formal thought disorder.

6

Page 7: Behavioral Science Questions

8/9/2019 Behavioral Science Questions

http://slidepdf.com/reader/full/behavioral-science-questions 7/53

A #D+year+old male is pleasant, emotionally warm, and happy. :e li(es with his parents, works in a sheltered

  workshop, did not complete high school and talks in word sentences. 3hich of the following is the most likely

  diagnosis4

  A. Autistic disorder 

  B. hildhood disintegrati(e disorder 

  . ental retardation

  ". i&ed recepti(e+e&pressi(e language disorder 

  %. =ett7s syndrome

  %&planation'

  The correct answer is . This young man is displaying simple mental retardation. :e is affable, interacts well

  with others to the point that he can work in a sheltered workshop, could not complete high school and talks in  three word sentences.

Autistic disorder *choice A- is not correct because persons with this diagnosis are rarely able to interact with  others to the point of holding a ob, and they characteristically do not use language for purposes of 

  communication.

  hildhood disintegrati(e disorder *choice B- is characteried by the clinically significant loss of pre(iously  ac9uired skills. This does not allow them to function well e(en in sheltered workshops.

i&ed recepti(e+e&pressi(e language disorder *choice "- is a neurologic condition and affects language skills.

  This man does ha(e both recepti(e and e&pressi(e language skills, which he can use to the point of being  producti(ely employed.

=ett7s syndrome *choice %- is associated with se(ere impairment, including loss of social engagement and

  pre(iously ac9uired *before the age of 0 months- skills, and characteristic hand wringing mo(ements. This  condition is confined to females. >ery recent e(idence suggests a genetic etiology for =ett7s syndrome.

3hat percentage of the population in the nited /tates had no health insurance co(erage of any type in 1<4

  A. 1E

  B. <E

  . 1<E

  ". $0E

  %. D0E

  %&planation'

D

Page 8: Behavioral Science Questions

8/9/2019 Behavioral Science Questions

http://slidepdf.com/reader/full/behavioral-science-questions 8/53

  The correct answer is . 1<E of the population of the nited /tates had no health insurance in 1<. 3hile

  people of lower economic status constitute much of this group, you should be aware that many employed,

  low+middle class to middle class indi(iduals also are uninsured, for reasons (arying from self+employment to

  pre+e&isting medical conditions. ost of the implications are ob(ious' less pre(entati(e care, less early  inter(ention in diseases, high costs to society at large.

A #+year+old male graduate student gets into a serious argument with one of his college professors, making a  physical threat to the professor, and necessitating a call to campus security. The argument was precipitated by

  an incident between the professor and the student7s girlfriend2 when the professor corrected the student7s

  girlfriend in class, the student felt the professor was (erbally abusi(e. 3hich of the following is the most likely

  diagnosis 4

  A. "ependent personality disorder 

  B. :istrionic personality disorder 

  . ;arcissistic personality disorder 

  ". )aranoid personality disorder 

  %. )assi(e aggressi(e personality disorder 

  %&planation'

  The correct answer is ". )ersons with this condition often percei(e attacks and danger in relati(ely innocuous  situations. They are 9uick to respond with anger, and, because personality disorders are ego+syntonic,

  indi(iduals with personality disorders do not belie(e themsel(es to be in error.

The indi(idual with dependent personality disorder *choice A- does not confront others but wants others to take  care of him.

The indi(idual with histrionic personality disorder *choice B- is flamboyant and seducti(e, not confrontational

  and angry.

The indi(idual with narcissistic personality disorder *choice - is characteried by feelings of entitlement

  because they are so special.

And the indi(idual with passi(e aggressi(e personality disorder *choice %- e&presses anger indirectly *e.g.,

  always being late- rather than confronting directly.

3hite coat hypertension is defined as an ele(ation of blood pressure resulting from the apprehension associated

  with (isiting the doctor. 8t is thought that the patient associates the physician7s white coat with distressing  e&periences *e.g., being (accinated as a child-, resulting in transient hypertension. This may be (iewed as a  physiological manifestation of which of the following phenomena4

A. lassical conditioning

  B. %&tinction

  . :abit hierarchies

!

Page 9: Behavioral Science Questions

8/9/2019 Behavioral Science Questions

http://slidepdf.com/reader/full/behavioral-science-questions 9/53

  ". ;egati(e reinforcement

  %. Operant conditioning

  %&planation'

  The correct answer is A. lassical conditioning in(ol(es the response toward one stimulus being transferred to

  another stimulus. 5or e&ample, a patient who fears going to the doctor e&periences heightened an&iety as the

  physician enters the room wearing a white coat. The patient7s fear then becomes associated with the white coat

  itself, such that future e&posure to this symbol e(okes similar apprehension in the patient.

%&tinction *choice B- means that when a beha(ior is no longer reinforced, it will disappear.

:abit hierarchies *choice - are ordered statements about the probability of occurrence of beha(iors. Those  beha(iors that ha(e been reinforced more strongly will be more likely to occur and will therefore be ranked

  higher in the response hierarchy.

 ;egati(e reinforcement *choice "- occurs when, in response to a beha(ior, an a(ersi(e condition is remo(ed

  rather than a positi(e reward being gi(en. 5or e&ample, a teenager may finally take out the garbage in order to  stop his mother from nagging him. This is a method in(ol(ed in operant conditioning *choice %-, which is based

  on the relationship between a response and the conse9uences *reinforcement- that follow that response.

A #D+year+old man has been arrested by the police for hitting, cursing at, and (erbally berating his wife of !

  years. The wife tells the police he also regularly physically whips his D+year+old son with a leather belt and often

  strikes the boy with his hand. 3hen asked why he does this, he responds that this is how my father treated me,  it7s how men should act. This represents which of the following types of learning4

  A. lassical conditioning

  B. ogniti(e learning

  . 8mprinting

  ". Operant conditioning

  %. /ocial learning

  %&planation'

  The correct answer is %. 8n social learning, also known as modeling, beha(ior is ac9uired by watching other 

  persons and assimilating their actions into the beha(ioral repertoire. There is no (erbal or cogniti(e process

  *choice B- that is in(ol(ed, no reinforcement *as in operant conditioning2 choice "-, no pairing of stimuli to get  stimulus substitution *as in classical conditioning2 choice A-, nor any early+life bonding or imprinting *choice -  in(ol(ed in this type of process. Because beha(iors such as spousal abuse, child abuse, and elder abuse are

  all based on obser(ing and incorporating beha(iors from significant others, the person displaying the beha(iors

  does not realie the beha(iors are inappropriate and is typically (ery resistant to change. The fact that the

  learning is non(erbal and not dependent upon reinforcement contributes to the resistance to change.

Page 10: Behavioral Science Questions

8/9/2019 Behavioral Science Questions

http://slidepdf.com/reader/full/behavioral-science-questions 10/53

A $#+year+old man has ust been informed that he has poorly differentiated small cell carcinoma of the lung.

  3hen asked if he understands the serious nature of his illness, the patient proceeds to tell his physician how

  e&cited he is about reno(ating his home. This patient is e&hibiting

  A. denial

  B. displacement

  . proection

  ". rationaliation

  %. reaction formation

  5. sublimation

  %&planation'

  The correct answer is A. This patient is in denial about his serious illness, and by talking about something  totally unrelated, he is trying to a(oid the bad news he has ust recei(ed.

"isplacement *choice B- in(ol(es the transferring of feelings to an inappropriate person, situation, or obect  *e.g., a man who has been yelled at by his boss takes out his anger on his wife-.

)roection *choice - is the attribution of one7s own traits to someone else *e.g., a philandering husband

  accuses his wife of ha(ing an affair-.

=ationaliation *choice "- in(ol(es creating e&planations for an action or thought, usually to a(oid self+blame.

=eaction formation *choice %- is the unconscious changing of a feeling or idea to its opposite *e.g., a man acts  (ery friendly toward a coworker when in fact he is unconsciously ealous-.

/ublimation *choice 5- in(ol(es turning an unacceptable impulse into an acceptable one *e.g., someone with

  (ery aggressi(e impulses becomes a professional bo&er-.

A <+year+old girl is brought by her parents to the emergency room because she is complaining of stomach pain.

  )hysical e&amination re(eals multiple bruises on the child7s body in different stages of healing. F+ray e&amination

  of the chest demonstrates two cracked ribs, and the child says, ommy hit me. The parents deny any abuse of 

  their children. The physician7s most appropriate response would be'

  A. 8 am going to call the police right now.

  B. 8 must report this situation to hild )rotecti(e /er(ices right now.

  . 8 need to hospitalie this child for further studies.

  ". 8 will bind her ribs tonight and you must promise me that you will not strike this child again.

  %. 8 will bind her ribs tonight and you need to bring her to the outpatient clinic in the morning.

10

Page 11: Behavioral Science Questions

8/9/2019 Behavioral Science Questions

http://slidepdf.com/reader/full/behavioral-science-questions 11/53

  %&planation'

  The correct answer is B. All signs, including the child7s report, suggest child abuse2 howe(er, there can be  mitigating circumstances that are present. All states ha(e laws re9uiring e(eryone to protect children by

  reporting the suspicion of child abuse to hild )rotecti(e /er(ices. 8t is the responsibility of this agency to

  pro(e or dispro(e the suspicion, and to establish super(ision of the child if abuse is (erified.

  8 am going to call the police right now *choice A-, is only appropriate if the hild )rotecti(e /er(ices is not

  a(ailable immediately.

hoices , ", and % do nothing to address the issue of the mandatory report of the suspicion of child abuse to  the appropriate authorities.

A #!+year+old male with history of mood disorder presents with a decreased need for sleep, irritability,

  recklessness, and increased energy. 3hich of the following is the most likely additional presenting symptom4

  A. "epressed mood

  B. 5ear of dying

  . 8nsomnia

  ". =acing thoughts

  %. =ecurrent thoughts and actions that relie(e an&iety when carried out

  %&planation'

  The correct answer is ". The presentation suggests a manic or hypomanic episode of a mood disorder.  )atients in a manic episode often ha(e an elated or euphoric mood and racing thoughts. Other symptoms of 

  mania include increased energy, hyperse&uality, grandiosity, and increased talkati(eness. 8n some patients,  irritability, rather than euphoria, is characteristic.

"epressed mood *choice A- can include anhedonia, decreased self+esteem, energy, concentration, appetite,

  and libido, as well as increased guilt and sucidality.

An unreasonable fear that one might die *choice B- may be associated with panic disorder. ;euro(egetati(e

  symptoms including se(ere an&iety, palpitation, shortness of breath, chest pain, trembling, and paresthesias

  may also occur.

"espite poor sleep, manic patients do not complain of insomnia *choice -. Their need for sleep is decreased.

  "uring depressi(e episodes, patients complain of insomnia or hypersomnia.

Obsessi(e+compulsi(e disorder is characteried by distressing recurrent thoughts *obsession- and actions

  *compulsion- *choice %- that relie(e an&iety when carried out.

A newborn has a heart rate of 10Gmin, irregular respirations, and acti(e muscle mo(ements with good tone. :e

  coughs and grimaces in response to stimulation2 he is pink in color, e&cept for his hands and feet, which are  slightly bluish. This neonate7s A)?A= score is

11

Page 12: Behavioral Science Questions

8/9/2019 Behavioral Science Questions

http://slidepdf.com/reader/full/behavioral-science-questions 12/53

  A. 6

  B. D

  . !

  ".

  %. 10

  %&planation'

  The correct answer is . A)?A= is an acronym for appearance, pulse, grimace, acti(ity, and respiration2 the  A)?A= score is taken at 1 minute and < minutes after birth. On each parameter, a ma&imum score of # is

  possible. 8n this case, one point was taken off for cyanosis of the hands and feet2 one point was taken off for 

  irregular respirations. The neonate recei(ed the ma&imum score of # for all of the other parameters, leading to

  an A)?A= of !.

A <+year old woman li(es alone. /he has ne(er been married, has a asters of Business Administration *BA-,  and is employed as a stock broker. /he was fired from her present firm because on three separate occasions

  o(er the last two years, without authoriation from her clients, she has sold all the securities in their accounts and

  in(ested the money in securities that had glossy portfolios, but were worthless. On these three separate

  occasions she has worked ##+#$ hours per day for 10 days at a time, gorged herself on unk food, and drank   alcohol e&cessi(ely. 3hich of the following is the most likely diagnosis4

  A. Bipolar disorder, type 8

  B. Bipolar disorder, type 88

  . yclothymic disorder 

  ". /chiophrenic disorder, paranoid type

  %. /ubstance+induced delirium

  %&planation'

  The correct answer is A. Bipolar disorder, type 8, is the appropriate diagnosis because she has had repeated

  manic episodes. The inappropriate grandiose acti(ity with her clients7 accounts *without the benefit of 

  consultation-, decreased need for sleep, and in(ol(ement in potentially self+destructi(e beha(ior *e.g.,  e&cessi(e alcohol consumption-, support this diagnosis.

  There is no history of depressi(e episodes, which is mandatory for the diagnosis of bipolar disorder, type 88

  *choice B-.

/ince her beha(ior is of psychotic proportion, and there is no history of depressi(e episodes, cyclothymic

  disorder *choice - is incorrect.

  )ersons with schiophrenic disorder, paranoid type *choice "-, ha(e a maor thought and affect disorder, and

1#

Page 13: Behavioral Science Questions

8/9/2019 Behavioral Science Questions

http://slidepdf.com/reader/full/behavioral-science-questions 13/53

  characteristically hallucinate. "uring an episode, they are unable to function in reality, e.g., selling and buying

  securities on the stock market. /he demonstrates no such beha(ior.

The hallmark of delirium *choice %- is a fluctuating le(el of consciousness. There are no indications in the  history that she is manifesting this symptom.

3hich of the following diseases should be reported to the "epartment of )ublic :ealth4

  A. andida albicans infection

  B. ondyloma acuminatum

  . ?onorrhea

  ". :8> infection

  %. /treptococcal pharyngitis

  %&planation'

  The correct answer is . ?onorrhea is on the short list of reportable diseases, including A8"/ *but not :8>

  positi(ity-, chickenpo&, hepatitis A and B, measles, mumps, rubella, salmonella, shigella, syphilis, and

  tuberculosis.

A 10+year+old boy was noted for his e&treme fear of water during his first day of swimming lessons. The teacher 

  helps the child sit on the edge of the pool and splash the water with his feet. /he then goes one step further and  shows him how to wet his knees. 8t takes her an hour to ha(e him float on the water while holding his hand. 3hat

  techni9ue has the teacher used to help the child con9uer his fear of water4

  A. lassical conditioning

  B. "esensitiation

  . %&tinction

  ". 5looding

  %. Operant conditioning

  %&planation'

  The correct answer is B. "esensitiation is an effecti(e therapy for phobia. The therapist models and guides the

  patient through progressi(e steps, starting with the least fearful step until the fear and an&iety associated with

  phobic obect is e&tinguished.

8n classical conditioning *choice A-, which was first described by )a(lo( in animal models, a neutral stimulus is

  paired with a stimulus that produces a response. The goal is to ha(e the neutral stimulus alone produce the  response. 8f food and the ringing of a bell produces sali(ation in a dog, repeated conditioning will cause

1

Page 14: Behavioral Science Questions

8/9/2019 Behavioral Science Questions

http://slidepdf.com/reader/full/behavioral-science-questions 14/53

  sali(ation in the dog upon hearing the bell alone.

The goal in e&tinction *choice - is to change a response by denying a reward that has maintained that

  response. 8f a child gets his parents7 attention by misbeha(ing, not paying attention to him can produce  e&tinction of the misbeha(ior.

8n flooding *choice "-, the therapist encourages the patient to confront the feared obect or situation without a

  gradual or graded e&posure.

8n operant conditioning *choice %-, the goal is to increase the likelihood of a response by reinforcement. A

  desired beha(ior is rewarded so it will be repeated and strengthened.

Of the following types of se&ual dysfunction, which would be the most difficult to treat4

  A. "yspareunia

  B. )remature eaculation

  . )rimary eaculatory incompetence in a male

  ". /econdary orgasmic dysfunction in a female

  %. >aginismus

  %&planation'

  The correct answer is . )rimary eaculatory incompetence means that the adult male has ne(er de(eloped the

  ability to eaculate while engaged in se&ual acti(ity with another person. 8t is e&tremely difficult to treat.

"yspareunia *choice A-, painful intercourse, is easily treated by addressing the underlying medical causes that

  e&ist in the (ast maority of cases.

)remature eaculation *choice B- is readily addressed by the s9ueee techni9ue.

/econdary orgasmic dysfunction *choice "- means there has been function but it has been lost. Any secondary

  se&ual dysfunction is easier to treat than a primary problem.

>aginismus *choice %- is often related to fear and apprehension regarding the act of penetration and is 9uite

  easily treated by beha(ior modification.

A 6$+year+old female is hospitalied for an acute e&acerbation of schiophrenia. The medications that she is  prescribed cause some immediate ad(erse effects, but she tolerates them and continues to be compliant. A few  years pass and she begins to de(elop neurologic abnormalities. These include in(oluntary, repetiti(e mo(ement

  of the lips and tongue, as well as of her trunk and e&tremities. 3hich of the following medications should now be

  prescribed to this patient4

  A. hlorpromaine

  B. loapine

1$

Page 15: Behavioral Science Questions

8/9/2019 Behavioral Science Questions

http://slidepdf.com/reader/full/behavioral-science-questions 15/53

  . 5luphenaine

  ". :aloperidol

  %. etoclopramide

  5. Thioridaine

  %&planation'

  The correct answer is B. A serious side effect of the antipsychotics is tardi(e dyskinesia, which has been seen

  with (irtually e(ery neuroleptic He.g., chlorpromaine *choice A-, fluphenaine *choice -, haloperidol *choice "-,

  and thioridaine *choice 5-I. sually, the symptoms of tardi(e dyskinesia appear late in treatment and consist of 

  in(oluntary, repetiti(e mo(ements of the lips and tongue *e.g., tongue thrusting, lip smacking-, and, not  infre9uently, of the e&tremities and trunk. )atients o(er 60 and those with pre+e&isting ;/ pathology are at a

  higher risk for this disorder *up to D0E-, but other risk factors ha(e not been confirmed. loapine is called an

  atypical antipsychotic medication because of its lack of e&trapyramidal side effects, including tardi(e dyskinesia,

  and would be an appropriate medication for a patient who is de(eloping tardi(e dyskinesia. etoclopramide

  *choice %- is a centrally acting antiemetic that has been shown to cause tardi(e dyskinesia as well.

A patient complains to her family physician that 3hen it7s time to go to work, 8 ust can7t seem to get out of the

  house. 8 ha(e a lot of windows, and 8 need to check them all three times. Then, 8 can ne(er be sure the door is

  locked, so 8 check it times. 87(e been late for work a few times, but this is the only way 8 can be sure the house is

  safe. /ometimes 8 think 87m going to go cray. 3hich of the following is the most likely diagnosis4

  A. Adustment disorder with an&iety

  B. Agoraphobia without history of panic disorder 

  . ?eneralied an&iety disorder 

  ". Obsessi(e compulsi(e disorder 

  %. )anic disorder with agoraphobia

  %&planation'

  The correct answer is ". This patient e&presses feelings of an&iety which are only relie(ed by compulsi(e ritual  beha(ior *checking the doors and windows e&cessi(ely-.

  Adustment disorder with an&iety *choice A- generally occurs in response to an identifiable stressor.

Agoraphobia without a history of panic disorder *choice B- is characteried by a fear of all large enclosed or 

  open spaces when alone.

  8n generalied an&iety disorder *choice -, e&cessi(e worry or an&iety is present most of the time the person is  awake, not ust in a specific situation.

8n panic disorder with agoraphobia *choice %-, panic attacks occur in uncued situations. This patient7s  symptoms appear consistently when she lea(es for work in the morning.

1<

Page 16: Behavioral Science Questions

8/9/2019 Behavioral Science Questions

http://slidepdf.com/reader/full/behavioral-science-questions 16/53

3hich of the following concepts is shared by 5reud, @ung, and urray4

  A. ollecti(e unconscious

  B. onditioning

  . "eath instinct

  ". Thematic Apperception Test

  %. nconscious

  %&planation'

  The correct answer is %. All three men shared the concept of the unconscious in their theories, although with a

  different emphasis. Through patient obser(ation and his own dream analysis, /igmund 5reud constructed a  system to e&plain that a patient7s needs and wishes e&ist in the unconscious mind. arl @ung e&panded 5reud7s

  concept and described a collecti(e unconscious as a symbolic and mythological past that is shared by all

  humankind. :enry urray de(eloped a thematic apperception test, a proecti(e techni9ue, to re(eal conscious  and unconscious mental processes.

As mentioned abo(e, arl @ung was the founder of the concept of collecti(e unconsciousness. @ung broke with

  5reud o(er the emphasis on infantile se&uality. :e e&panded the concept of indi(idual unconscious to collecti(e  unconscious *choice A-.

B. 5. /kinner de+emphasied the unconscious and demonstrated, through e&perimental analysis, that

  personality is the result of reinforcement and conditioning *choice B-.

"eath instinct *choice - was the term that 5reud used in 1#0 before designating aggression as a separate  instinct. :e regarded the life and death instincts as forces that underlie se&ual and aggressi(e instincts. Today,

  most clinical phenomena can be e&plained by se&uality and aggression instincts without recourse to the  concept of death instinct.

urray emphasied that moti(ation leads to continuous acti(ity until the need is reduced or satisfied. :e

  de(eloped the Thematic Apperception Test *choice "- to re(eal personal and interpersonal conflicts, needs,  and attitudes.

A #<+year+old male presents to his family physician with the following statement' "octor, 8 can7t urinate in public

  restrooms. 8 can if there is no one around, but if 8 go to the restroom in a mo(ie, an airport, at the ball park or 

  anywhere that someone else comes in, 8 can7t urinate. %(en if 8 ha(e already started, it ust stops and 8 can7t get it  going until the other person lea(es. 8 am so embarrassed. 3hat do they think of me if they see 8 can7t do what  e(ery other man can do4 3hich of the following is the most likely diagnosis4

  A. An&iety disorder due to a general medical condition

  B. )anic disorder 

  . /ocial phobia

16

Page 17: Behavioral Science Questions

8/9/2019 Behavioral Science Questions

http://slidepdf.com/reader/full/behavioral-science-questions 17/53

  ". /pecific phobia

  %. /ubstance+induced an&iety disorder 

  %&planation'

  The correct answer is . 8n the condition described, a person is in a social situation and fears that he or she will

  not be able to perform in the same manner as most e(eryone else can. The two most common social phobias

  concern public speaking and restroom performance *sometimes called shy bladder.-

An&iety disorder due to a general medical condition *choice A- is diagnosed when a medical condition

  precipitates an&iety, e.g., hypoglycemia.

)anic disorder *choice B- is characteried by sudden paro&ysms of an&iety. 8t can strike une&pectedly in uncued  situations so it would not occur only when others are present.

/pecific phobias *choice "- are unreasonable fears of some identifiable thing, not situation *e.g., ele(ators-.

  /ubstance+induced an&iety disorder *choice %- is diagnosed when an&iety is precipitated by ingestion of a  psychoacti(e substance, e.g., hallucinogens.

A #$+year+old secretary is walking home from work late one night and is accosted by a man with a ski mask who

  robs her. /i& weeks later, the woman still has recurrent, intrusi(e thoughts about the e&perience, and states that

  she feels numb and is easily startled whene(er she sees a stranger on the street. Three months after the  incident, the symptoms ha(e largely subsided. 8n all likelihood, this patient will e&perience which of the following4

  A. ?radual deterioration in functioning

  B. ;o recurrence of symptoms

  . )ersistent flashbacks

  ". =ecurrent depression

  %. >isual and auditory hallucinations

  %&planation'

  The correct answer is B. The woman described was suffering from acute posttraumatic stress disorder. This

  condition is triggered by e&posure to a traumatic e(ent in(ol(ing the threat of death or serious bodily harm to

  oneself or others, especially in a situation in which the person is helpless andGor intensely fearful. Acute  posttraumatic stress disorder is characteried by symptom duration of less than months, and is generally  followed by a full reco(ery.

  Acute posttraumatic stress disorder is not accompanied by a gradual deterioration in functioning *choice A-, as

  is seen in psychotic states and dementia.

  )ersistent flashbacks *choice - are associated with chronic posttraumatic stress disorder, which is

  characteried by symptoms lasting more than months. The chronic form carries a much worse prognosis than  acute posttraumatic stress disorder.

1D

Page 18: Behavioral Science Questions

8/9/2019 Behavioral Science Questions

http://slidepdf.com/reader/full/behavioral-science-questions 18/53

  =ecurrent depression *choice "- is not associated with uncomplicated acute posttraumatic stress disorder.

  >isual and auditory hallucinations *choice %- may occur with acute posttraumatic stress disorder, but would not  be e&pected to continue beyond months. 8f this did occur, the diagnosis of chronic posttraumatic stress

  disorder should be considered.

A +year+old woman who was herself a (ictim of child abuse becomes the primary caregi(er for her elderly

  mother. %(entually, the daughter begins abusing her mother. This is an e&ample of what type of learning4

  A. lassical conditioning

  B. ogniti(e learning

  . 8mprinting

  ". Operant conditioning

  %. /ocial learning

  %&planation'

  The correct answer is %. /ocial learning is that type of interpersonal ac9uisition of beha(iors accomplished by

  watching a model perform the acti(ity. 3hen a stronger person models abuse of a weaker person, the weaker   person often learns that as an appropriate response for later in their life.

  lassical conditioning *choice A- is most associated with ac9uisition of beha(iors by simple contiguity, and

  operates mainly in learning associated with the autonomic ner(ous system.

ogniti(e learning *choice B- focuses on purposeful understanding and full cogniti(e awareness of information  ac9uisition.

8mprinting *choice - is not correct because this is the type of learning that is operati(e in bonding, in which

  one person *usually an infant- ac9uires attachment to another person *usually the mother- simply by being

  associated. 8t is central to affiliati(e processes, not to the disaffiliation one sees in elder abuse.

Operant conditioning *choice "- is not correct because this type of learning is dependent upon a reward or 

  reinforcement being pro(ided for a gi(en beha(ior. %lder abuse is not a beha(ior that is systematically trained

  into the abuser.

A D+year+old woman is abducted, beaten, and repeatedly raped. 5or fi(e months after the attack she is  ner(ous, tearful, easily fatigued, and has difficulty concentrating. /he also notes difficulty sleeping and lack of   appetite, and is hyperreacti(e to une&pected or loud stimuli. By si& months she has returned to her characteristic

  pre+attack beha(ioral patterns. 3hich of the following is the most likely diagnosis4

  A. Acute stress disorder 

  B. Adustment disorder with mi&ed an&iety and depressed mood

1!

Page 19: Behavioral Science Questions

8/9/2019 Behavioral Science Questions

http://slidepdf.com/reader/full/behavioral-science-questions 19/53

  . aor depressi(e disorder 

  ". )anic disorder without agoraphobia

  %. )ost+traumatic stress disorder 

  %&planation'

  The correct answer is B. The reaction was precipitated by a stressful e(ent that would cause anyone to

  e&perience a se(ere and intense emotional response. The symptoms occurred within months of the e(ent

  and lasted for less than 6 months after the trauma, all corresponding to the diagnosis of adustment disorder   with mi&ed an&iety and depressed mood.

  Acute stress disorder *choice A- and post+traumatic stress disorder *choice %- both re9uire that the person

  ree&perience the traumatic e(ent in wakeful or dream states and the presence of dissociati(e symptoms.

This is not a maor depressi(e disorder *choice - because her symptoms are not of psychotic proportion and

  there is a clear precipitating stimulus.

)anic disorder without agoraphobia *choice "- is incorrect because the panic episodes are uncued and not the  response to an en(ironmental stressor.

A 10+year+old girl who is a suspected (ictim of child abuse is referred to a psychologist for e(aluation. As part of 

  her workup, the patient is asked to construct a story based on pictures. 3hich of the following psychometric

  measures was utilied4

  A. innesota ultiphasic )ersonality 8n(entory

  B. yers+Briggs )ersonality 8n(entory

  . =orschach Test

  ". Thematic Apperception Test

  %. Type A and B Beha(ior )atterns Test

  %&planation'

  The correct answer is ". The Thematic Apperception test is a proecti(e test employing pictures depicting  ambiguous interpersonal situations that the e&aminee is asked to interpret. )sychodynamic theory suggests

  that since the stimuli are (ague, the patient proects his or her own thoughts, feelings, and conflicts into his or 

  her responses, pro(iding the e&aminer insight into the patient7s thought and memory content.

The innesota ultiphasic )ersonality 8n(entory *)82 choice A-, which uses true and false items, is the most

  popular obecti(e personality test.

The yers+Briggs )ersonality 8n(entory *choice B- is based on @ungian theory and assesses basic dimensions  of personality *e&tro(ersion-2 it is used e&tensi(ely in occupational counseling. The patient selects preferred

  adecti(es from groups of choices.

The =orschach Test *choice - is another proecti(e test that in(ol(es asking patients to describe what they

1

Page 20: Behavioral Science Questions

8/9/2019 Behavioral Science Questions

http://slidepdf.com/reader/full/behavioral-science-questions 20/53

Page 21: Behavioral Science Questions

8/9/2019 Behavioral Science Questions

http://slidepdf.com/reader/full/behavioral-science-questions 21/53

  on(ersion disorder *choice A- is manifested by chronic neurologic pain or deficit without any obecti(e organic

  cause. This patient7s chief complaint is acute pain without any sign of another neurologic deficit.

  "iffuse muscle and oint aches, in conunction with lack of energy and reduced physical acti(ity, is characteristic

  of depression *choice B-. This patient7s pain is localied and she is (ery acti(e.

  =uptured or inured tendon of the ankle is a common inury, but the patient does not recall any recent inury,  and localiation of the pain is not consistent with medial ankle inury *choice -.

  The girl7s daily walking in the nearby forest raises suspicion for tick bite *choice %-and Cyme disease with

  secondary oint pain. Arthritis associated with Cyme disease is generally centered around oints, and does not  present as acutely as in this patient.

1.

  Acceptance

#.

  Anger

.  Bargaining

$.

  "enial<.

  /adnessGdepression

3hich of the abo(e correctly e&presses the order of Jubler+=oss7s stages of dying4

  A. 1+#+$+<+

  B. #+<++1+$

  . +#+$+1+<

  ". $+#++<+1

  %. <++$+1+#

  %&planation'

  The correct answer is ". 8f you understand the thought process behind these stages, you are unlikely to forget  the progression. )atients first e&perience denial that they are dying. pon awareness, they become angry at

  their lot and proceed to bargain for a more fa(orable outcome. 3hen this pro(es futile, they become

  depressed. Then, they finally accept the truth. The mnemonic is "AB/A.

A #+year+old male re(eals to his psychiatrist that he has been hearing (oices telling him to kill his girlfriend.

  3hich of the following principles re9uires that the physician inform the girlfriend that she is in danger4

A. ?ood /amaritan law

#1

Page 22: Behavioral Science Questions

8/9/2019 Behavioral Science Questions

http://slidepdf.com/reader/full/behavioral-science-questions 22/53

  B. 8rresistible 8mpulse rule

  . c;aughten rule

  ". Tarasoff 8 decision

  %. Tarasoff 88 decision

  %&planation'

  The correct answer is ". The Tarasoff 8 decision re9uires that physicians warn a potential (ictim if they truly  belie(e the patient will harm the potential (ictim.

The ?ood /amaritan law *choice A- states that stopping at the scene of an accident to render care is not

  re9uired but that once physicians initiate such care they must practice within their confidence and may not  abandon the patient.

The 8rresistible 8mpulse rule *choice B- is in(ol(ed in insanity defenses in criminal prosecution. 8t considers

  whether a person7s actions were under (oluntary control or resulted from an uncontrollable passion.

The c;aughten rule *choice - is also in(ol(ed in insanity defenses. 8t acknowledges that people may not

  realie the nature and conse9uences of their actions because of mental illness.

The Tarasoff 88 decision *choice %- states that e(en though physicians must warn a potential (ictim, they must

  also protect the patient from harm from the potential (ictim.

A D+year+old, left+handed boy is brought to the family physician for a preschool physical e&amination and

  immuniations. pon e&amination, the physician finds bruises, in different stages of healing, all o(er the child7s

  body, including his chest and back. :e also notes cigarette burns on his left arm, and bruises on his scrotum.

  The boy7s mother states he is (ery clumsy and falls all the time, he burns himself with cigarettes, and  masturbates constantly. The most likely e&planation for these findings is

  A. autistic disorder in the child

  B. borderline personality disorder in the child

  . child abuse syndrome in the parent*s-

  ". de(elopmental coordination disorder in the child

  %. factitious disorder in the parent*s-

  %&planation'

  The correct answer is . The e&planations pro(ided by the mother do not make logical sense' e.g., the boy is

  left+handed and his left arm is burned, male masturbation does not include s9ueeing the scrotum hard enough

  to bruise it, and there are multiple bruises in different stages of healing. =egarding the latter symptom, children

  do fall and run into things, resulting in bruising. The bruises children usually obtain, howe(er, are on the lateral  aspects of the arms, the shins, and upper thighs. Bruising on the chest and back in different stages of healing

  is suspect.

  Autistic disorder *choice A- re9uires some suggestion of interpersonal social withdrawal or communication

##

Page 23: Behavioral Science Questions

8/9/2019 Behavioral Science Questions

http://slidepdf.com/reader/full/behavioral-science-questions 23/53

  disorder.

  The diagnosis of borderline personality disorder *choice B- *or any other personality disorder- re9uires that the

  patient is at least 1! years of age.

  "e(elopmental coordination disorder *choice "- is characteried by impairment in the ability to carry out daily

  acti(ities to the point that normal functioning is impaired.

  5actitious disorder *by pro&y- in the parents *choice %- is incorrect since the inuries to the child are not se(ere

  enough to get the child into the sick role and pro(ide the parents with primary gain.

A <<+year+old male begins group therapy. After the first session, he befriends one of the other clients, and

  begins telling her how e&traordinarily intelligent and talented the facilitator is. At the ne&t session, he and the  facilitator disagree. After the session, he tells his fellow group member that the facilitator is utterly incompetent

  and that they should sue for malpractice. This is an e&ample of

A. displacement

  B. fi&ation

  . reaction formation

  ". regression

  %. splitting

  %&planation'

  The correct answer is %. /plitting is a primiti(e defense mechanism in which obects or people are thought of as  either all bad or all good. This defense mechanism is normal in young children, but also occurs in adults with

  borderline personality disorder *the man in group therapy- or psychosis.

"isplacement *choice A- is an unconscious defense mechanism in which one7s feelings or desires are

  unconsciously transferred from their original obect to a more acceptable substitute.

  5i&ation *choice B- refers to an arrest of de(elopment at a particular de(elopmental stage. 8t is generally a

  partial or incomplete arrest of de(elopment, but can contribute to the de(elopment of emotional problems if 

  protracted.

  =eaction formation *choice - refers to an unconscious defense mechanism in which the person takes on an

  attitude or belief that is the opposite of his or her true beliefs and desires.

  =egression *choice "- is a return to an earlier *often infantile- stage of de(elopment that occurs in many mental  illnesses and in normal indi(iduals e&periencing tragic or e&tremely stressful e(ents.

A $+year+old male presents to the hospital complaining of weight loss, nausea, (omiting, and lethargy. There is

  no e(idence of edema, orthostatic hypotension, or dehydration. Blood samples show hyponatremia, while his

  urine is highly concentrated. A water+load test is performed, in which the patient is instructed to drink a large  (olume of water and < hourly samples of urine are analyed. All samples show concentrated urine. 3hich of the

#

Page 24: Behavioral Science Questions

8/9/2019 Behavioral Science Questions

http://slidepdf.com/reader/full/behavioral-science-questions 24/53

  following drugs could ha(e caused this condition4

  A. Baclofen

  B. arbamaepine

  . "antrolene

  ". )heneline

  %. )henytoin

  5. Tranylcypromine

  %&planation'

  The correct answer is B. The patient abo(e is suffering from syndrome of inappropriate secretion of A":

  */8A":-. 3hen A": *(asopressin- is secreted in e&cessi(e amounts, or at inappropriate times, it can cause

  hyponatremia as well as symptoms of nausea, (omiting, anore&ia, and lethargy. Also, e&cessi(e release of   (asopressin results in the e&cretion of a concentrated urine *with a urinary osmolality usually o(er 00 mmolGkg-

  despite a subnormal plasma osmolality and serum sodium concentration. /8A": can be caused by many

  factors, such as ectopic A": production and release from neoplastic tissue *small cell carcinoma of lung,  pancreatic carcinoma, lymphosarcoma, :odgkin7s disease, reticulum cell sarcoma, thymoma, and carcinoma of 

  duodenum or bladder- or by drugs that release or potentiate the action of A":, such as carbamaepine,

  (incristine, (inblastine, cyclophosphamide, chlorpropamide, general anesthetics, and tricyclic antidepressants.

Baclofen *choice A- is a spasmolytic agent. Baclofen has not been shown to be associated with the release of 

  A":.

"antrolene *choice - is a spasmolytic agent and is also used to treat malignant hyperthermia. "antrolene has  not been shown to be associated with the release of A":.

)heneline *choice "- is an AO inhibitor. AO inhibitors ha(e not been shown to be associated with the

  release of A":.

)henytoin *choice %- is an antiseiure drug. )henytoin has not been shown to be associated with the release of 

  A":.

Tranylcypromine *choice 5- is an AO inhibitor. AO inhibitors ha(e not been shown to be associated with the

  release of A":.

A $!+year+old male is brought to the psychiatric emergency room after an attempted suicide. :e claims to hear 

  (oices telling him to kill himself. The patient7s family notes that he has been on se(eral different kinds of   antipsychotic medications, with no impro(ement of his symptoms. The attending psychiatrist places the patient on  a new medication, and admits him. One week after therapy has begun, a routine blood test re(eals profound

  depletion of polymorphonuclear leukocytes. 3hich of the following drugs is most likely responsible for these

  symptoms4

  A. hlorpromaine

  B. loapine

#$

Page 25: Behavioral Science Questions

8/9/2019 Behavioral Science Questions

http://slidepdf.com/reader/full/behavioral-science-questions 25/53

  . 5luo&etine

  ". :aloperidol

  %. 8mipramine

  5. )heneline

  %&planation'

  The correct answer is B. loapine is an antipsychotic drug that has been shown to cause agranulocytosis.

  Agranulocytosis is an acute condition characteried by pronounced leukopenia, with great reduction in

  polymorphonuclear leukocytes *K <00 cells per mm-. 8nfected ulcers are likely to form in the throat, intestinal

  tract, and other mucous membranes as well as on the skin. Other side effects caused by cloapine include  orthostatic hypotension, sinus tachycardia, hypersali(ation, temperature ele(ation, lowered seiure threshold,

  and constipation. loapine is generally prescribed only after se(eral other alternati(e neuroleptic medications

  ha(e failed, because of the possibility of agranulocytosis and the drug7s prohibiti(e cost.

  hlorpromaine *choice A- is an antipsychotic drug that has antimuscarinic side effects such as dry mouth and  constipation. 8t can also cause orthostatic hypotension, sedation, and tardi(e dyskinesia. 8t does not cause

  agranulocytosis.

  5luo&etine *choice - is a selecti(e serotonin reuptake inhibitor *//=8-, an antidepressant drug that can cause

  an&iety and insomnia, altered appetite and weight loss, acti(ation of mania or hypomania, seiures, and

  cogniti(e motor impairment.

:aloperidol *choice "- is an antipsychotic drug that has less antimuscarinic side effects than does

  chlorpromaine, but has more e&trapyramidal effects, such as acute dystonia *face, neck, and back 

  spasmsLndash2abnormal posture-, parkinsonism, neuroleptic malignant syndrome *catatonia, rigidity, stupor,

  fe(er, dysarthria, fluctuating B)-, and akathisia *restlessness-. 8t can also cause tardi(e dyskinesia. 8t does not  cause agranulocytosis.

8mipramine *choice %- is a tricyclic antidepressant drug that can cause orthostatic hypotension, anticholinergic

  effects, antihistamine effects, and hypomania. 8t does not cause agranulocytosis.

)heneline *choice 5- is an AO inhibitor that can cause orthostatic hypotension *once the body adapts to

  higher basal le(els of catecholamines, it is no longer able to further (asoconstrict in response to stress-,

  hepatoto&icity, and hypomania. 8t does not cause agranulocytosis. =emember the risk of de(eloping a  hypertensi(e crisis when taking an AO inhibitor with tyramine+containing foods such as wine and aged

  cheeses.

A man brings his $<+year+old wife to the emergency department. :e states she has been ill for days and has  been running a fe(er of .! to 100.<M 5. Today she is ha(ing difficulty staying awake, is talking to persons who

  are not there, and at times appears to be frightened of something. /he is restless and somewhat combati(e

  when restrained. 3hat is the most likely diagnosis4

  A. Acute stress disorder 

  B. Bipolar 8 disorder, manic type

  . Brief psychotic disorder 

  ". "elirium

#<

Page 26: Behavioral Science Questions

8/9/2019 Behavioral Science Questions

http://slidepdf.com/reader/full/behavioral-science-questions 26/53

  %. "ementia

  %&planation'

  The correct answer is ". This is a psychotic le(el disorder *the patient is hallucinating-, she has a fluctuating

  le(el of consciousness, and she is disoriented. Also, there is a clear history of a febrile condition that  de(eloped rather rapidly, all of which suggest delirium.

  8n acute stress disorder *choice A- a traumatic e(ent occurs that precipitates an an&iety+type reaction, not a

  change in the sensorium.

  8n both bipolar 8 disorder, manic type *choice B- and brief psychotic disorder *choice -, patients may reach a

  le(el of beha(ioral disruption of psychotic proportion. They do not, howe(er, demonstrate changes in le(el of 

  consciousness or maor disorientation.

  )ersons with dementia *choice %- demonstrate a clear sensorium with no fluctuations in the le(el of 

  consciousness. Additionally, persons with dementia predominantly show symptoms of impairment of cogniti(e

  functions *e.g., memory impairment-.

A 1<+year+old girl is brought to the emergency room by concerned friends who said she was acting (ery erratic.  :er friends belie(e that she had been drinking or taking drugs. /he is agitated, ata&ic, and disoriented. /e(eral

  people are re9uired to hold her down for a physical e&amination that re(eals tachycardia, hypertension, normal

  bowel sounds, mydriasis, and nystagmus. 3hich of the following drugs of abuse is most likely responsible for her 

  symptoms4

  A. Amphetamine

  B. %thanol

  . Cysergic acid diethylamide

  ". :eroin

  %. )hencyclidine

  %&planation'

  The correct answer is %. )hencyclidine *))-, also known as angel dust, is a dissociati(e anesthetic that can  be taken orally, by smoking, or by intra(enous inection. )) causes disorientation, detachment, reckless

  beha(ior, impaired udgement, and distortions of body image. /omatic signs include horiontal or (ertical

  nystagmus, hypertension, tachycardia, diaphoresis, motor incoordination, and numbness. :igh doses can  produce (omiting, seiures, stupor, coma, or death.

  Amphetamine *choice A- produces euphoria, ner(ousness, hyperacti(ity, anore&ia, short attention span,

  mydriasis, tachycardia, hypertension, sweating, and insomnia. hronic use can cause symptoms similar to

  paranoid schiophrenia. Amphetamine does not cause nystagmus.

  %thanol *choice B- produces ata&ia, psychomotor impairment, and disinhibition. Acute alcohol into&ication would

  not be e&pected to cause tachycardia, hypertension, mydriasis, or nystagmus.

#6

Page 27: Behavioral Science Questions

8/9/2019 Behavioral Science Questions

http://slidepdf.com/reader/full/behavioral-science-questions 27/53

  C/" *choice - produces perceptual distortions but few obser(able beha(ioral changes. /omatic symptoms

  include nausea, weakness, and paresthesias.

  :eroin *choice "- and other opiates produce constricted pupils, a lethargic or semi+somnolent state,  hypotension, and decreased bowel sounds.

A 1+year+old girl is brought to the emergency room by the police. /he had run away from home after another 

  battle with her mother. /he has been hospitalied se(eral times for o(erdoses, and she has numerous scars on

  her wrists. The psychiatrist notes that all of her relationships ha(e been stormy, and that she seems to regard

  people as either all good or all bad. /he is admitted with a diagnosis of maor depression because of the  apparent depth of her depression, howe(er, by the ne&t morning, she is completely reco(ered and is well with

  no (egetati(e symptoms. 3hich of the following is the most likely diagnosis4

  A. Antisocial personality disorder 

  B. Borderline personality disorder 

  . :istrionic personality disorder 

  ". ;arcissistic personality disorder 

  %. )assi(e+aggressi(e personality disorder 

  %&planation'

  The correct answer is B. Borderline personality disorder is characteried by short term psychotic episodes *e.g.,

  the depression noted in this case-, self mutilation, splitting persons into the good+bad e&tremes on a

  continuum, and markedly unstable interpersonal relationships.

  The person with antisocial personality disorder *choice A- operates in opposition to society7s rule and customs.  riminal beha(ior is common.

The person with histrionic personality disorder *choice - is flamboyant and seducti(e.

The person with narcissistic personality disorder *choice "- is impressed with himself and operates from a

  position of entitlement.

  The person with passi(e+aggressi(e personality disorder *choice %- e&presses his anger by passi(e means

  such as procrastination, chronic tardiness, and sabotaging producti(ity.

8n a restaurant, a +year+old child screams shrilly as loudly as she can. The mother gi(es the child a piece of   cookie each time she screams. 3hich of the following types of reinforcement does the mother recei(e when the

  child stops screaming4

  A. A(ersi(e

  B. 5i&ed inter(al

#D

Page 28: Behavioral Science Questions

8/9/2019 Behavioral Science Questions

http://slidepdf.com/reader/full/behavioral-science-questions 28/53

  . ;egati(e

". )ositi(e

%. >ariable =atio

  %&planation'

  The correct answer is . 3hen the child takes away the shrill screaming *which was a(ersi(e to the mother- it is

  reinforcing to the mother and guarantees that the mother will continue the beha(ior *gi(ing a piece of cookie-,

  which will guarantee that the child will continue to scream. /ince the reinforcing e(ent is the remo(al of an  a(ersi(e stimulus, this is called negati(e reinforcement.

A(ersi(e reinforcement *choice A- in(ol(es doing something that the child would not like, e.g., gi(ing a

  spanking.

  5i&ed inter(al reinforcement *choice B- implies that a gi(en amount of time goes by before reinforcement is

  a(ailable.

  There is no positi(e reinforcement *choice "- for the mother, because nothing that she wants is being gi(en.  3hat is reinforcing to her is the fact that something she does not want is being taken away.

  A (ariable ratio *choice %- reinforcement schedule means that the child would only stop screaming sometimes  when gi(en a cookie *on a schedule that the mother could not predict-.

A #0+year+old woman sees her baby cousin for the first time. As she attempts to play with the infant, he begins to

  cry incessantly. :ow old is this baby likely to be4

  A. 1+$ months

  B. <+! months

  . +1# months

  ". 1+16 months

  %. 1D+#0 months

  %&planation'

  The correct answer is B. The baby is e&hibiting stranger an&iety, which normally occurs between the ages of <  and months.

Cet7s re(iew some other social milestones that are good to be aware of during clinical work in pediatrics'

/pontaneous smiling begins within se(eral days after birth and disappears by months. /miling at any face  occurs by # months, followed 9uickly by smiling only at familiar faces and when pleased. By months, infants

  can imitate facial e&pressions. They laugh at $ months.

rying occurs from birth. 8t peaks at 6 weeks and is most fre9uent from $+6 p.m. olic is defined as crying more

#!

Page 29: Behavioral Science Questions

8/9/2019 Behavioral Science Questions

http://slidepdf.com/reader/full/behavioral-science-questions 29/53

  than hours a day for more than days a week. 8t often spontaneously resol(es by $ months. Treatment

  includes holding, a(oiding o(erstimulation, and antispasmodics.

/eparation an&iety occurs between the ages of 10 and 1! months, when the infant is separated from the  mother.

Between the ages of # months and # years, children might show preference for a comforting transitional

  obect *e.g., teddy bear-, which is usually discarded by age $, when the transition from dependence on the  mother to independence is more complete.

A #D+year+old male is brought into the emergency room by the police, who found him walking aimlessly, shouting

  the names of former )residents. rine to&icology is negati(e, and the man appears to be oriented with respect to

  person, place, and time. :e has had fi(e similar admissions o(er the past year. Attempts to inter(iew the patient

  are fruitless, as he seems easily derailed from his train of thought. A phone call to a friend listed in the chart  pro(ides the additional information that the man is homeless, and unable to care for himself. This patient is

  e&hibiting the signs and symptoms of

A. schioaffecti(e disorder 

  B. schioid personality disorder 

  . schiophrenia

  ". schiophreniform disorder 

  %. schiotypal personality disorder 

  %&planation'

  The correct answer is . The patient is suffering from schiophrenia. The key to the diagnosis of psychosis is  that there has been a marked decline in the le(el of functioning *i.e., the man is homeless and cannot care for 

  himself-. Although hallucinations or delusions are not mentioned in the case history, the presence of   disorganied speech, grossly disorganied beha(ior, and the duration of symptoms *longer than si& months-

  suggest a diagnosis of schiophrenia.

8n schioaffecti(e disorder *choice A-, alterations in mood are present during a substantial portion of the illness.

Although schioid personality disorder *choice B- produces detachment from social relationships and is

  characteried by restriction of emotional e&pression, it is not accompanied by a marked decline in occupational

  functioning.

/chiophreniform disorder *choice "- is characteried by schiophrenic+like symptoms, but the duration of 

  symptoms is, by definition, less than si& months.

/chiotypal personality disorder *choice %- is characteried by eccentricities of beha(ior, odd beliefs or magical

  thinking, and difficulties with social and interpersonal relationships. nlike schiophrenia, schiotypal personality

  disorder is not characteried by a formal thought disorder.

A $+year+old patient is e(aluated for suicidal ideation after he is found laying on train tracks by police. The man

  is dishe(eled and malodorous and states that he has reached the end and would rather die. :e admits to  depressed mood, anhedonia, poor energy and appetite2 he feels miserable and regrets what he has done with

#

Page 30: Behavioral Science Questions

8/9/2019 Behavioral Science Questions

http://slidepdf.com/reader/full/behavioral-science-questions 30/53

  his life and wants to put an end to it. :e states he has felt this way since age #6, after he was discharged from

  the military. The man indicates that his life was great until he increased his drinking, which caused a di(orce at

  age 0. :e has had two arrests for dri(ing under the influence. :e was in ail for 6 months after he had an

  accident while drunk that resulted in public property damage. :e remembers that he initially felt sick in ail, with  sweating, (omiting, shaking, and he e&perienced a seiure. :e then impro(ed after a few days and felt better 

  during the rest of his imprisonment without any depression. 3hich of the following criteria most strongly suggests

  alcohol abuse4

  A. "esire to cut down

  B. =ecurrent drunk dri(ing

  . /eiure after withdrawal

  ". /uicidal ideation

  %. Tolerance

  %&planation'

  The correct answer is B. The "/ criteria for alcohol abuse are recurrent use resulting in failure to fulfill

  obligations, recurrent use in haardous situations, recurrent legal problems related to use, and continued use  despite negati(e conse9uences. 8t is also important to note that the patient has ne(er met the criteria for 

  dependence.

  A persistent desire or unsuccessful efforts to cut down *choice A- is also a symptom of dependence.

  /eiures *choice - are a symptom of se(ere withdrawal in chronic alcoholics. 3hen the patient de(elops

  withdrawal symptoms and tolerance, re9uiring larger amount to achie(e the desired effect, he or she has met

  the criteria for dependence.

  /uicidal ideation *choice "- in this patient is a conse9uence of his depression, which in turn, is secondary to  alcohol use. ontinued use of a substance despite knowledge of persistent or recurrent physical or 

  psychological problems is also a criterion for dependence.

  Tolerance *choice %- is defined as the need to increase substance use to achie(e the desired effect, or 

  diminished effect with continuous use of the same amount. Tolerance is a maor criterion of dependence.

After his spouse dies from ad(anced malignant melanoma, a man comes to his family physician stating that he is  e&periencing a great deal of guilt about his wife7s death and feels that he could ha(e done more to sa(e her.

  3hich of the following is the best response the physician can make to this patient at this time4

  A. "on7t talk like that.

  B. 8 think e(eryone goes at their appointed time.

  . ;obody could ha(e sa(ed her.

  ". Tell me more about your feelings of guilt.

0

Page 31: Behavioral Science Questions

8/9/2019 Behavioral Science Questions

http://slidepdf.com/reader/full/behavioral-science-questions 31/53

  %. Nou did the best you could.

  %&planation'

  The correct answer is ". 3hen a person loses someone who is close to them the most important thing they

  need to do is to talk about the loss. Any response that does anything other than allow and encourage the

  person to (erbalie their feelings is inappropriate. Tell me more about your feelings of guilt is the only  statement that encourages the patient to talk.

The other choices are all statements that will dissuade the person from talking more about the loss.

A 0+year+old heterose&ual male repeatedly cross+dresses to achie(e se&ual e&citement, but is content with his

  biological gender. 3hat is the most likely diagnosis4

  A. %&hibitionism

  B. Transse&ualism

  . Trans(estic fetishism

  ". >oyeurism

  %&planation'

  The correct answer is . "o not confuse trans(estic fetishism with transse&ualism *choice B-. Transse&uals

  might cross dress but they do so due to persistent discomfort with their anatomic se&. Trans(estic fetishism

  in(ol(es no such discomfort.

  %&hibitionism *choice A- refers to e&posing one7s genitals to unsuspecting strangers for se&ual e&citement.

>oyeurism *choice "- refers to obser(ing unsuspecting people, generally strangers, who are naked,  undressing, or are engaged in se&ual acti(ity, for se&ual e&citement.

A 60+year+old male e&ecuti(e with a history of angina pectoris and depression had bypass surgery the pre(ious

  day. :is depression has responded well to selecti(e serotonin reuptake inhibitors *//=8s- and there is no history

  of psychosis in the past. :e now presents with confusion, agitation, irritability, and tries to remo(e his 8> lines. :is

  le(el of consciousness fluctuates, and at times he forgets who he is. :e is gi(en a neuroleptic drug, and appears  much impro(ed. 3hat is the most likely diagnosis4

  A. Adustment disorder 

  B. "elirium

  . "ementia

  ". %&acerbation of depression with suicidal ideation and psychotic features

  %. /chiophrenia

1

Page 32: Behavioral Science Questions

8/9/2019 Behavioral Science Questions

http://slidepdf.com/reader/full/behavioral-science-questions 32/53

  %&planation'

  The correct answer is B. "elirium is a common complication of general anesthesia and surgery. 8t is manifested

  by acute changes in mental status with wa&ing and waning le(el of consciousness, agitation, irritability, and

  psychosis. )atients usually respond to low+dose neuroleptics to achie(e sedation. The course is self+limited.

  Any psychosocial or biological stressor can lead to adustment disorder *choice A-. This patient7s surgery will

  restrict his le(el of functioning, at least in the short term. This will be difficult for a high+functioning indi(idual to

  accept. Adustment disorder may present with depressi(e mood, an&iety, and irritability, but a fluctuating le(el of 

  consciousness is not a feature of this disorder.

"ementia *choice - can present with irritability, confusion, and agitation, but usually has an insidious course

  and affects mainly cognition. 8n contrast to delirium, it does not ha(e a fluctuating course.

  /e(ere depression can present with irritability, suicidal ideation, and psychotic features *choice "-. The patient

  has a history of depression that responded well to //=8s and he has no prior history of psychosis. :e was

  moti(ated to undergo cardiac surgery, so remo(ing his 8> lines is unlikely to be a manifestation of suicidal

  ideation.

  /chiophrenia *choice %- presents with biarre beha(ior, hallucinations, and delusions. 8t usually starts at a

  younger age than the acute symptoms in this patient, and is characteried by progressi(e deterioration in

  functioning. 8t is unlikely for a schiophrenic to achie(e the functional le(el of an e&ecuti(e.

A 1D+year+old girl loses her best friend in an automobile accident. After the death, she starts writing for hours  daily in her diary. This would most likely be an e&ample of which of the following defense mechanisms4

  A. 8dentification

  B. )roection

  . =ationaliation

  ". =egression

  %. /ublimation

  %&planation'

  The correct answer is %. /ublimation is the di(ersion of unacceptable impulses into more acceptable ones. 8n

  this case, the girl would like to continue to talk to her best friend, but since that is not possible, she substitutes

  writing in her diary. An e&ample of sublimation seen fairly fre9uently in medical settings is the mother whose  child died of a disease who becomes acti(e in a state or national organiation designed to help families with the  disease.

  8dentification *choice A- is the adoption of characteristics or acti(ities of another person.

  )roection *choice B- occurs when someone attributes their own thoughts to a different person.

  =ationaliation *choice - is the offering of a false, but acceptable, e&planation for beha(ior.

#

Page 33: Behavioral Science Questions

8/9/2019 Behavioral Science Questions

http://slidepdf.com/reader/full/behavioral-science-questions 33/53

  =egression *choice "- is the adoption of beha(ior more appropriate to a younger age.

3hat is the earliest age at which toilet training is likely to be successful4

  A. 10 months

  B. 1 months

  . 16 months

  ". 1 months

  %. ## months

  %&planation'

  The correct answer is ". Toilet training is not possible before the age of 1! months because the long ner(e  fibers ha(e not yet myelinated and sphincter control is not possible. Toilet training should be completed by $

  years of age.

An !<+year+old man presents with complaints of pain in his left chest on inspiration. )hysical e&amination re(eals

  bilateral bruises on his upper arms. F+ray films of his chest show three broken ribs on the left side. The most

  likely e&planation for this constellation of findings is

  A. alcoholic incoordination

  B. elder abuse

  . falling in the bathtub

  ". phase 888 Alheimer7s disease

  %. physical se9uela of pseudodementia

  %&planation'

  The correct answer is B. The bilateral bruises on the upper arms suggest that he has been tightly grabbed. The  left+sided rib fractures would support the possibility that he was struck forcefully by someone who is

  right+handed *as most people are-.

Alcoholic incoordination *choice A- characteristically results in bruises on the lateral surface of the body as the  person stumbles into door frames, or on the shins at coffee table height.

5alling in the bathtub *choice - is likely to produce bruises localied to one side of the body.

There are no characteristic physical signs of trauma associated with either Alheimer7s disease *choice "- or 

  pseudodementia *choice %-.

Page 34: Behavioral Science Questions

8/9/2019 Behavioral Science Questions

http://slidepdf.com/reader/full/behavioral-science-questions 34/53

Page 35: Behavioral Science Questions

8/9/2019 Behavioral Science Questions

http://slidepdf.com/reader/full/behavioral-science-questions 35/53

  The correct answer is A. The patient described in this 9uestion is suffering from absence seiures, which

  typically appear during childhood, between the ages of < and D. 8n absence seiures, the patient has many

  episodes of brief disruption of consciousness throughout the day. These seiures are not accompanied by the  con(ulsions and complete loss of consciousness often associated with epilepsy, but rather by the absence of 

  motor or sensory symptoms *hence the blank look on the patient7s face-.

hildren with attention deficit hyperacti(ity disorder *choice B- also ha(e a limited attention span and normal  intelligence. :owe(er, they also e&hibit hyperacti(ity, impulsi(eness, emotional lability, and irritability, which lead

  to beha(ioral problems in school.

hildren with infantile autism *choice - may present with a short attention span, but their most striking deficits  lie in their difficulty with social interactions and communication skills. 8nfantile autism is a de(elopmental disorder 

  that usually manifests itself before age , and most autistic children ha(e an 8 below the normal range.

)honological disorders *choice "- are a class of communication disorders in which the age+ and  intelligence+appropriate speech sounds are de(elopmentally delayed.

  /chiophrenia with childhood onset *choice %- is 9uite rare. hildren with this disorder demonstrate normal

  intelligence, and may show a limited attention span. :owe(er, these children also manifest the same psychiatric

  symptoms seen in adult+onset schiophrenics, including hallucinations, delusions, abnormal affect, and limited  social skills, which lead to beha(ioral problems in school.

A $$+year+old male with a history of polysubstance abuse presents with nausea, (omiting, increased heart rate,

  high blood pressure, sweating, agitation, and weakness. :e also complains of seeing monsters on the wall during

  the inter(iew. 3hich of the following best accounts for this presentation4

  A. Alcohol into&ication

  B. Alcohol into&ication or cocaine withdrawal

  . Alcohol withdrawal

  ". Alcohol withdrawal or cocaine into&ication

  %. ocaine into&ication

  5. ocaine withdrawal

  %&planation'

  The correct answer is ". This man7s presentation can be e&plained either by alcohol withdrawal or by cocaine

  into&ication. ;ausea, (omiting, sympathetic ner(ous system acti(ation, and weakness could be produced in

  either case. >isual hallucinations *seeing monsters- can also be accompanied by tactile and auditory  hallucinations. 8n se(ere cases, either alcohol withdrawal or cocaine into&ication can cause con(ulsions.

Alcohol into&ication *choice A and B- is characteried by disinhibition, aggression, impaired attention and

  udgment, unsteady gait and imbalance, slurred speech, nystagmus, and a decreased le(el of consciousness.

  Other symptoms associated with alcohol withdrawal *choices and "- include insomnia, headache, and

  tremors of the tongue, eyelids, and outstretched hands.

  3ithdrawal from cocaine *choices B and 5- is characteried by dysphoria, lethargy, psychomotor retardation or 

<

Page 36: Behavioral Science Questions

8/9/2019 Behavioral Science Questions

http://slidepdf.com/reader/full/behavioral-science-questions 36/53

  agitation, increased appetite, sleep disturbances, and biarre or unpleasant dreams.

  Other symptoms associated with cocaine into&ication *choices " and %- include grandiosity, paranoid ideation,

  and pupillary dilatation. Orientation usually remains intact.

A 6D+year+old woman who has been in (ery good health is brought to her physician7s office by her husband. :e

  states that o(er the course of the last < years she has had difficulty recogniing her grandchildren, he must do all

  the planning for their daily acti(ities, she forgets that she has things cooking on the sto(e, and at night he

  sometimes finds her wandering through the house with an absent look on her face. /he is beginning to  demonstrate difficulty in recalling the names of common obects, and her speech is limited to simple two+ or 

  three+word sentences. 3hich of the following is the most likely diagnosis4

  A. Alheimer7s disease

  B. Amnestic disorder 

  . )seudodementia

  ". /ubstance+induced persisting dementia

  %. >ascular dementia

  %&planation'

  The correct answer is A. This woman is suffering from dementia of the Alheimer7s type. A gradual onset of 

  symptoms, general per(asi(e memory deficit, difficulties with language, and inability to plan, leading to se(ere

  impairment of daily functioning are all characteristic of dementia of the Alheimer7s type.

Amnestic disorder *choice B- is limited to memory problems and this woman is demonstrating cogniti(e  dysfunction, such as alterations in language and the loss of the ability to plan.

  )seudodementia *choice - is incorrect since it is a maor depressi(e disorder rather than a dementing

  condition. There is no e(idence for a depressi(e syndrome in this patient7s presentation.

  The diagnosis of substance+induced persisting dementia *choice "- re9uires e(idence of a history of substance  abuse. :owe(er, it is the second most likely diagnosis and should be carefully e&plored with the husband and

  other close relati(es and friends.

  >ascular dementia *choice %- is generally characteried by a step+wise deterioration, not the gradual  presentation of this case.

A <+year+old widowed female is brought to the emergency room by her family after they noticed increasing  irritability, agitation, and abusi(eness. /he recently had a loud altercation with a new neighbor. :er past history is

  significant for depression, which was treated with paro&etine for $ years. =ecently, the woman has been staying

  up all night doing housework, and denies feeling tired the ne&t day. /he recently surprised a family friend with

  se&ually inappropriate, seducti(e remarks. /he denies any hallucinations at the present time, but acknowledges  that she has heard (oices in the past, telling her to kill herself. /he currently denies suicidal ideation and states

  that life is ust great e&cept that she is worrying about her grandchildren while she is in the hospital. 3hich of 

  the following is the most likely diagnosis4

6

Page 37: Behavioral Science Questions

8/9/2019 Behavioral Science Questions

http://slidepdf.com/reader/full/behavioral-science-questions 37/53

  A. Adustment disorder 

  B. An&iety disorder 

  . ood disorder 

  ". )ersonality disorder 

  %. Thought disorder 

  %&planation'

  The correct answer is . The patient has a history of depression. /he now presents with symptoms of mania,

  including increased goal+directed acti(ity, possible hyperse&uality *seducti(e remarks-, irritability, and  decreased need for sleep. 3hile the primary diagnosis has been unipolar depression, the current presentation

  is consistent with bipolar disorder *manic+depressi(e-. Both depression and bipolar affecti(e disorder are mood

  disorders.

A healthy indi(idual should be able to adust to new conditions such as a new neighbor, but the patient is clearly  e&hibiting symptoms of an affecti(e disorder, rather than an adustment disorder *choice A-.

An&iety disorder *choice B- is characteried by e&cessi(e worrying. This alone does not e&plain the current  presentation2 an&iety disorder can occur simultaneously with mood disorder, or as part of it.

  )ersonality disorders *choice "- are diagnosed when maladapti(e and rigid traits in an indi(idual produce

  distress andGor functional impairment2 these traits are usually stable and predictable. )ersonality disorders are  classified as a&is 88 in "/ 8>. "iagnosis of a&is 88 is usually deferred until the patient7s a&is 8 disorder *the mood

  disorder- is stabilied.

The patient has history of auditory hallucinations, which suggest the presence of a thought disorder *choice %-  such as schiophrenia. :owe(er, mood disorders can present with psychotic features. This patient heard (oices

  telling her to kill herself, probably during a period of se(ere depression2 these hallucinations were congruent  with her likely mood at the time, e(idence that they were part of the underlying affecti(e disorder.

A 1+year+old mother brings her first child, a 10+day+old infant, to the pediatrician. 8n a fearful tone of (oice she  states' %(ery time 8 drop something or the dog barks, or if 8 turn the lights on, he umps and erks his little arms

  to his chest like he7s afraid. 8s something wrong with him4 The pediatrician e&plains that the beha(ior is normal

  and is called the

  A. Babinski refle&

  B. deep tendon refle&

  . oro refle&

  ". palmar refle&

  %. tonic neck refle&

D

Page 38: Behavioral Science Questions

8/9/2019 Behavioral Science Questions

http://slidepdf.com/reader/full/behavioral-science-questions 38/53

  %&planation'

  The correct answer is . The oro refle& can be elicited in the infant by any startling e(ent2 it consists of 

  e&tension and abduction of the arms, followed by fle&ion and adduction of the arms. This is a normal refle& that  appears between the ages of #< and 6 weeks of gestation, and will normally disappear between +6 months.

  The Babinski refle& *choice A- is elicited when the lateral surface of the sole of the foot is stroked resulting in

  the great toe going up and the other toes fanning. 8t normally disappears at 1 year of age.

  "eep tendon refle&es *choice B- can be elicited by tapping a tendon with a refle& hammer, stretching the

  tendon and producing contraction in the corresponding muscle. These are present throughout life.

  The palmar grasp refle& *choice "- is characteried by the infant7s hand closing o(er an obect that is placed in

  the palm of the hand. This refle& normally disappears at # months of age.

  The tonic neck refle& *choice %- consists of e&tension of the ipsilateral leg and fle&ion of the contralateral arm  and leg when the head is turned. This refle& normally disappears between D and ! months of life.

A $0+year+old woman is being seen by a physician for the 10th time this year for e(aluation of (ague aches and  pains. An e&tensi(e prior e(aluation has e&cluded the possibility of serious disease. "uring the inter(iew with this

  patient, she makes repeated statements along the lines of, 3hat 8 want doesn7t matter. "o what you want. And,

  87m afraid you won7t ha(e time to see me anymore. The traits this patient is e&hibiting are most consistent with  which of the following personality disorders4

  A. "ependent

  B. :istrionic

  . Obsessi(e+compulsi(e

  ". )aranoid

  %. /chioid

  %&planation'

  The correct answer is A. This scenario is classic for dependent personality. Cook for reliance on others,

  subordination of own needs, and fear of abandonment. ;ote that in real life, patients may show symptoms of 

  more than one personality disorder.

  :istrionic personality disorder *choice B- is characteried by theatricality, suggestibility, a strong desire for 

  attention, and shallowness.

  Obsessi(e+compulsi(e personality disorder *choice -, also called anancastic personality disorder, is  characteried by obsessions, perfectionism, rigidity, and self+doubt.

  )aranoid personality disorder *choice "- is characteried by suspiciousness, o(ersensiti(ity, 9uerulousness,

  and an unforgi(ing character.

  /chioid personality disorder *choice %- is characteried by emotional coldness, solitude, and social insensiti(ity.

!

Page 39: Behavioral Science Questions

8/9/2019 Behavioral Science Questions

http://slidepdf.com/reader/full/behavioral-science-questions 39/53

8n which of the following ways does the sleep pattern of a D!+year+old differ from that of a younger indi(idual4

  A. 8ncreased need for sleep

  B. 8ncreased =% sleep

  . ore arousal and awakening at night

  ". ore total nighttime sleep

  %. /ignificant sleep disturbances are more common

  %&planation'

  The correct answer is . /leep patterns differ from person to person, howe(er some generaliations can be

  made regarding age and sleep. %lderly indi(iduals ha(e more awakening and arousal at night, they tend to

  awaken earlier, and ha(e less total sleep. ultiple factors can contribute to sleep disturbances in the elderly,

  including primary sleep disorders, sleep disorders secondary to other physical and psychiatric conditions, as  well as medication+induced sleep problems.

  3ith increased age, the needs for sleep decreases *compare with choice A-. A newborn might sleep up to ##  hours daily, but an adult can generally feel rested with 6 to ! hours of sleep.

  The amount of time spent in =% sleep decreases with age *compare withchoice B-, starting at about age <0.

  The elderly generally achie(e less total nighttime sleep *compare with choice "-.

  ild sleep disturbances *compare with choice %- can be associated with normal aging, howe(er any significant

  sleep disturbance that impairs daily acti(ity or causes increased daily sleepiness re9uires further e(aluation.

A #$+year+old female is brought to the emergency room after threatening to kill herself by cutting her wrists. /he  has multiple scars on her wrists, which she admits were caused by prior suicide attempts. /he states she is (ery

  angry at her boyfriend, who left her for another woman. /he pre(iously thought her boyfriend was an angel and

  now she thinks he is a monster. /he feels (ery empty inside. 3hile smiling, she states that she is depressed.

  "uring the inter(iew, she drops to the ground, but continues to talk while lying on the floor. /he belie(es nobody  understands her. 3hat is her underlying personality disorder4

  A. Antisocial

  B. Borderline

  . :istrionic

  ". ;arcissistic

  %. /chioid

  %&planation'

Page 40: Behavioral Science Questions

8/9/2019 Behavioral Science Questions

http://slidepdf.com/reader/full/behavioral-science-questions 40/53

  The correct answer is B. haracteristics of borderline personality disorder include frantic beha(ior to a(oid

  abandonment, unstable interpersonal relationships, alternating between idealiation and de(aluation *splitting-,

  recurrent suicidal gestures or other types of self+mutilatory beha(ior, feelings of emptiness, and inappropriate

  intense anger.

  An antisocial patient *choice A- does not confirm to social norms, is deceitful, impulsi(e, reckless, irresponsible,

  and lacks remorse for wrongdoings.

  One symptom that would suggest histrionic personality disorder *choice - in this patient is her theatrical

  e&aggeration of her emotions by talking while lying on the floor. A histrionic patient might present with

  attention+seeking andGor seducti(e and pro(ocati(e beha(ior, but the presence of splitting, recurrent suicidal

  gestures and anger argue strongly for the diagnosis of borderline personality disorder.

  A narcissistic patient *choice "- is grandiose and preoccupied with success, feels special and re9uires

  admiration, feels entitled, takes ad(antage of others, lacks empathy, and is arrogant.

  A schioid patient *choice %- is usually not interested in relationships or pleasurable acti(ities, is a loner, lacks

  friends, is emotionally cold, and is indifferent to praise or criticism.

An oncologist tells his patient that her laboratory results support a diagnosis of ad(anced malignant melanoma

  with multiple metastases to the li(er and brain. :e also ad(ises her that the prognosis is poor. 3hich of the

  following is most likely to be the first statement that the patient will make4

  A. an you keep me ali(e until my daughter graduates from medical school4

  B. "amn you doctor, you should ha(e caught this earlierP

  . "octor, you must be wrong.

  ". 8 think it is time that 8 make a will and say good+bye to e(eryone.

  %. 8t7s no use, 8 always lose and get the short end of the stick.

  %&planation'

  The correct answer is . Jubler+=oss7s death and dying se9uence is a step+wise process with < identified  stages. The order in which these stages appear is the following' 1. denial, #. anger, . bargaining, $. sadness,

  and <. acceptance. "octor you must be wrong is the correct answer since it reflects the patient7s inability to

  accept the information and indicates the denial of the first stage.

  an you keep me ali(e until my daughter graduates from medical school *choice A- is a statement from the

  rd, bargaining stage.

  "amn you doctor, you should ha(e caught this earlier *choice B- is a statement from the #nd or anger phase.

8 think it is time that 8 make a will and say good+bye to e(eryone *choice "- reflects the patient7s acceptance of 

  the reality and is a statement from the <th phase *acceptance-.

8t7s no use, 8 always lose and get the short end of the stick *choice %- is a statement from the $th phase

  *sadness-.

$0

Page 41: Behavioral Science Questions

8/9/2019 Behavioral Science Questions

http://slidepdf.com/reader/full/behavioral-science-questions 41/53

A 1+year+old male is brought to the emergency room by the authorities after the man7s mother, with whom he

  li(es, called them for assistance. %arlier in the day the young man began to shout that they were after him, and

  stated he was going to kill the sons+a+bitches before they got him. :e also was talking to persons who were not  there, became (ery agitated and finally threatened his mother7s life. 8n the emergency room he is shouting,

  combati(e, abusi(e and threatening to e(eryone who comes close to him. 3hich of the following is the most likely

  diagnosis4

  A. Bipolar 8 disorder, most recent episode manic

  B. "elusional disorder

. )aranoid personality disorder 

  ". /chiophrenic disorder, paranoid type

  %. /chioaffecti(e disorder 

  %&planation'

  The correct answer is ". /chiophrenia is a disease of young adults with the initial onset usually in the

  mid+to+late teens. The young man is psychotic, as defined by the presence of hallucinations, which rules out a  personality disorder *choice -. :is condition is characteried by delusions of persecution and hallucinations.

  Bipolar disorder with a manic episode *choice A- is not manifested by paranoid delusions and is usually

  accompanied by euphoria, not the ob(ious dysphoria this man is demonstrating. /uch patients are rarely  threatening to others or self during their manic episode.

"elusional disorder *choice B- is characteried by non+biarre delusions of at least one month7s duration.

  Additionally, persons with this diagnosis usually do not ha(e hallucinations unless they are integral to the  delusional content.

  /chioaffecti(e disorder *choice %- re9uires the presence of symptoms of schiophrenia and a maor affecti(e

  disorder. This man does not demonstrate the latter, as there is no apparent mood disorder.

A #+year+old man li(ing in a group home for the de(elopmentally challenged has an 8 of D. :e does not read  or write, and communicates with one or two word utterances. :e will not interact with other group home members,

  and since birth he has pulled back and becomes agitated when others get physically close to him. 8n his room,

  e(erything is in a gi(en place2 if any of his belongings are mo(ed, he becomes 9uite disturbed. 3hich of the

  following is the most likely diagnosis4

  A. Asperger7s disorder 

  B. Autistic disorder

. hildhood disintegrati(e disorder 

  ". Obsessi(e compulsi(e disorder

%. /chiophrenia, catatonic

$1

Page 42: Behavioral Science Questions

8/9/2019 Behavioral Science Questions

http://slidepdf.com/reader/full/behavioral-science-questions 42/53

  %&planation'

  The correct answer is B. The man is displaying the classic signs of autism, which include withdrawal from  interaction with others, failure to use speech for communication, and the obsessi(e need for sameness.

  %(en though patients with Asperger7s disorder *choice A- display a social interaction deficit, there is no

  language delay. Also, stereotyped patterns of beha(ior *e.g., hand twisting- occur in this disorder.

  hildren with childhood disintegrati(e disorder *choice - de(elop normally for the first two years of life, and

  then demonstrate deterioration, but this patient has demonstrated pathology from birth.

  Obsessi(e compulsi(e disorder *choice "- does not include withdrawal from physical contact, language

  impairment, or presence from birth.

  /chiophrenic disorder, catatonic type *choice %-, has its onset in late adolescence, and is characteried by  difficulties in mo(ement, and either immobility or e&citement. The patients also are negati(istic and demonstrate

  biarre posturing.

A pathologist recei(es a phone call from a patient who was diagnosed with a 1<+cm recurrent, mediastinal

  seminoma. "uring the con(ersation it becomes clear that despite the patient7s ob(ious intelligence, the man does

  not really understand that his disease is probably incurable. The patient keeps talking about his long+term plans  for the future, and belie(es that the pathologist must ha(e Lld9uo2misunderstoodLrd9uo2 how big his tumor was.

  Cater, the surgeon tells the pathologist that he has spent 1# hours o(er the last si& months trying to e&plain the

  prognosis to the patient. The patient is most likely using which of the following defense mechanisms4

  A. "enial

  B. "isplacement

  . =eaction formation

  ". =egression

  %. =epression

  %&planation'

  The correct answer is A. This is denial, in which a person beha(es as if he or she is unaware of something he

  may reasonably be e&pected to know. "enial is common in medical settings and this case is a real one. "enial is  distinguished from the related concept of repression mostly by the deeper le(el of subconsciousness at which

  the latter occurs. )eople with deeply repressed memories do not usually try to argue with someone who talks to

  them2 they simply do not remember.

  "isplacement *choice B- is a transfer of emotion from one setting to another.

  =eaction formation *choice - is the unconscious adoption of beha(ior opposite to one7s true feelings and

  intentions.

  =egression *choice "- is the adoption of beha(ior appropriate to an earlier stage of de(elopment.

  =epression *choice %- is the deeply subconscious suppression of traumatic e(ents or thoughts.

$#

Page 43: Behavioral Science Questions

8/9/2019 Behavioral Science Questions

http://slidepdf.com/reader/full/behavioral-science-questions 43/53

A $+year+old girl is disco(ered pulling the tail of the family dog. :er mother, who had warned her pre(iously that  this beha(ior was unacceptable, now assigns the girl to a < minute time out period. A timer is set up so that the

  girl can keep track of the time period. After two minutes of the period, the girl begins to scream and cry. At this

  point, the mother7s best response would be to

  A. do nothing until the time out period has ended

  B. e&plain to the child why this type of punishment is necessary

  . offer the child a choice between becoming 9uiet or being paddled

  ". reset the timer to add an additional < minutes to the time out period

  %. tell the child that if she 9uiets down she will be rewarded by a treat at the end of the time out period

  %&planation'

  The correct answer is ". The purpose of the time out is to remo(e the girl from stimuli to facilitate the e&tinction

  of unwanted beha(ior. To be effecti(e, time out must be used consistently and predictably. 8f the child protests  during a time out, as in this case, additional time is added to the period to e&tinguish the protest beha(iors. The

  goal is to con(ey the clear message that the time out will be ended only when unacceptable beha(iors are

  ended.

To impact beha(ior, the inter(ention must be closely associated with the beha(ior. To delay in responding to

  the girl7s protests *choice A- makes it harder for her to realie that the additional time is a direct conse9uence

  of her beha(ior.

  At age , reasoned, rational e&planations *choice B- are unlikely to ha(e any impact on the child7s beha(ior.

  The child probably lacks the cogniti(e capacity to grasp the abstract rationale for why she is being disciplined.

  )addling *choice - is a type of attention, and can actually reinforce the beha(ior the parents are trying to  e&tinguish. The child learns that protestation will bring the parents and their attention, e(en if it is negati(e

  attention. 8n addition, as a general rule, any response option on the /C% e&am that has someone hitting a

  child will almost certainly be scored as a wrong answerP

  This option encourages the child to cry and then demand a treat to become 9uiet *choice %-, a kind of u(enile

  blackmail. A(oiding bad beha(ior is a baseline and need not be specially rewarded.

The parents of a <+year+old girl ask their family physician for ad(ice regarding their child. The mother had walked

  into the girl7s bedroom without knocking and disco(ered the child stimulating her genitals. The parents are

  concerned, but seem to be recepti(e. The best response the physician could gi(e is'

  A. "o you think that someone7s been molesting her4

  B. "on7t you think you should knock before going into her room4

  . /he probably has a (aginal infection. Bring her in so 8 can e&amine her.

  ". This is perfectly normal beha(ior for a child this age.

$

Page 44: Behavioral Science Questions

8/9/2019 Behavioral Science Questions

http://slidepdf.com/reader/full/behavioral-science-questions 44/53

  %. 3hat disturbs you about this beha(ior4

  %&planation'

  The correct answer is %. Before the physician can pro(ide guidance for the parents, the parents7 concerns

  need to be understood. 3hile the described beha(ior is perfectly normal for a <+year+old *choice "-, and it is  appropriate for parents to knock on the door of their child7s room before entering *choice B- to teach children

  respect for pri(acy through modeling, the parents7 concerns must first be understood.

  To immediately assume there is something physically wrong with the child *choice - or that the child has been  se&ually abused *choice A- suggests that the physician may ha(e some personal issues with children7s normal

  se&uality.

A #D+year+old swimmer who feels insecure about her athletic abilities harshly criticies her teammates7

  techni9ues. 3hich of the following ego defense mechanisms is she displaying4

  A. "isplacement

  B. )roection

  . =eaction formation

  ". =epression

  %. /ublimation

  %&planation'

  The correct answer is B. )roection in(ol(es attributing one7s own traits, feelings, and attitudes to someone  else. This #D+year+old swimmer7s harsh criticism of her teammates7 abilities is a reflection of her personal

  feeling of incompetence. *"oubts about her own ability are translated into doubts about her teammates7  aptitude.-

"isplacement *choice A- in(ol(es the automatic transferring of a wish or an affect from one obect to a

  substitute. 5or e&ample, a man who is angry at his wife releases his hostility by kicking the table.

=eaction formation *choice - in(ol(es turning a repressed impulse or unconscious wish to its opposite. 5or 

  e&ample, a man who is attracted to his brother7s wife de(elops an a(ersion to her personality.

=epression *choice "- occurs when the conflicting thought or feeling is automatically hidden from the person7s

  awareness. 5orgetting an emotionally charged e(ent is an e&ample of repression.

/ublimation *choice %- is a (ery mature mechanism that in(ol(es consciously turning socially unacceptable  impulses into acceptable or more benign forms. 5or e&ample, a young college girl immerses herself in athletics

  rather than engage in premarital se&.

A <#+year+old white male who had been found wandering the streets is brought into the hospital by the police. On

$$

Page 45: Behavioral Science Questions

8/9/2019 Behavioral Science Questions

http://slidepdf.com/reader/full/behavioral-science-questions 45/53

  initial physical e&am, his motor beha(ior is notable for bradykinesia and a $+6 : hand tremor at rest. :e is kept

  under obser(ation in the psychiatric ward, but is not medicated. O(er the ne&t few days, his motor symptoms start

  to abate, but he becomes increasingly paranoid and confused, and he insists that he is the )resident of the

  nited /tates. 3hich of the following conditions best describes the patient at the time of admission4

  A. Alcoholic suffering from acute symptoms of withdrawal

  B. hronic amphetamine user suffering from drug+induced psychosis

  . hronic schiophrenic suffering from tardi(e dyskinesia

  ". )arkinsonian patient o(ermedicated with C+dopa

  %. /chiophrenic o(ermedicated with haloperidol

%&planation'

  The correct answer is %. The patient is a schiophrenic o(ermedicated with haloperidol. 3hen the patient is first

  brought into the hospital, he is suffering from )arkinsonian motor symptoms that are a significant side effect of   many neuroleptics *particularly haloperidol-. O(er the ne&t few days, he remains unmedicated, and the effects

  of haloperidol begin to wear off, which relie(es his motor symptoms, but leads to the reappearance of his

  psychotic symptoms.

3hile alcohol withdrawal *choice A- can produce delirium tremens, it would not e&plain the initial presentation

  with )arkinsonian symptoms.

hronic amphetamine use *choice B- can result in an amphetamine+induced psychosis that resembles an acute

  schiophrenic attack. :owe(er, these attacks abate within a few days after drug use ceases. This patient7s

  psychosis surfaced after a few days without medication.

hronic schiophrenics *choice - with an e&tensi(e history of neuroleptic use can de(elop tardi(e dyskinesia,

  which is characteried by in(oluntary aw and tongue mo(ements.

A )arkinsonian patient o(ermedicated with C+dopa *choice "- may suffer from (isual and auditory hallucinations  as well as in(oluntary mo(ements. These symptoms are the result of increased acti(ity in the dopamine system,

  and would be e&pected to abate after se(eral days without treatment. )arkinsonian motor symptoms repressed

  by C+dopa would be e&pected to re+emerge as the drug is cleared from the system.

A child psychiatrist would like to e(aluate the intellectual ability of a +year+old patient. 3hich of the following is

  the most appropriate test for him to use4

A. "en(er "e(elopmental /cale

  B. /tanford+Binet /cale

  . 3A8/+= 

  ". 38/ 888

  %. 3))/8

$<

Page 46: Behavioral Science Questions

8/9/2019 Behavioral Science Questions

http://slidepdf.com/reader/full/behavioral-science-questions 46/53

  %&planation'

  The correct answer is B. The /tanford+Binet scale is best for younger children *#+$ years old-, since it does not  rely e&clusi(ely on language.

The "en(er "e(elopmental /cale *choice A- is used to assess the attainment of de(elopmental milestones in

  children under #.

The 3A8/+= *3echsler Adult 8ntelligence /cale2 choice - is used for indi(iduals aged 1D and o(er. *@ust think,

  the 3A8/+= is rated =-.

The 38/ 888 *3echsler 8ntelligence /cale for hildren2 choice "- is useful for e(aluating children aged 6+16.

The 3))/8 *3echsler )reschool and )rimary /cale of 8ntelligence2 choice %- is used for children aged $+6.

A +year+old woman is brought into the emergency room by ambulance. /he has been diagnosed as ha(ing

  schiophrenic disorder, disorganied type, since the age of 1D. /he has been on antipsychotic medications since

  that time, which ha(e controlled her symptoms well. )hysical e&amination re(eals a well+nourished female with a  temperature of 10.# degrees 5, B) of 1!0G, := of D, and copious perspiration. /he is mute, has muscular 

  rigidity and appears to be obtunded. 3hich of the following is the most likely diagnosis4

  A. Acute dystonia

  B. Akathisia

. ;euroleptic malignant syndrome

  ". )arkinsonism

  %. Tardi(e dyskinesia

  %&planation'

  The correct answer is . ;euroleptic malignant syndrome *;/- is a potentially fatal condition that can occur at

  any time during the course of treatment with neuroleptics. The e&act etiology is unknown. %&cessi(e muscle  contraction produces muscular rigidity, and is also responsible for the high temperature. The obtunded mental

  state and mutism is characteristic. uscle rela&ants, such as dantrolene, and dopamine agonists, such as

  bromocriptine, are used in the treatment of ;/.

  Acute dystonia *choice A, prolonged contractions of muscle groups-, akathisia *choice B, restless legs -, and

  parkinsonism *choice ", pill+rolling tremor and rigidity- are all e&trapyramidal side effects that occur early during

  neuroleptic treatment.

  Tardi(e dyskinesia *choice %- is a late+appearing complication of neuroleptic therapy characteried by perioral

  and athetoid mo(ements.

An adolescent male is referred for drug abuse. :e confides to the therapist that he has been taking large

  amounts of reds *secobarbital sodium- for some time, and that they make him feel confident and calm. :e would  like to try to stop and says he would like to do it on his own, without additional medication. The therapist should

$6

Page 47: Behavioral Science Questions

8/9/2019 Behavioral Science Questions

http://slidepdf.com/reader/full/behavioral-science-questions 47/53

  ad(ise the patient to deto&ify with medical assistance because of the danger of 

  A. insomnia

  B. rebound an&iety

  . recidi(ism

  ". respiratory depression

  %. seiures

  %&planation'

  The correct answer is %. /ecobarbital is a short+acting barbiturate with considerable dependence potential.

  3ithdrawal from short+acting barbiturates can produce an&iety, delirium, and seiures which may be

  accompanied by life+threatening cardio(ascular collapse.

  8nsomnia *choice A- is a complication of barbiturate withdrawal, since barbiturates are sedati(eGhypnotic agents,  but this complication is not serious enough to be a contraindication to abrupt cessation of the drug.

=ebound an&iety *choice B- would be 9uite likely following abrupt cessation of the barbiturate, but would not  constitute a sufficient danger to the patient to preclude self+deto&ification.

=ecidi(ism *choice - is 9uite likely in drug abusers, with or without medical inter(ention.

=espiratory depression *choice "- is common with acute administration of barbiturates, but would not be

  e&pected with barbiturate abstinence.

A child who understands that the (olume of a li9uid poured out of a narrow glass remains the same when poured  into a wider glass is at which of )iaget7s stages of intellectual de(elopment4

  A. oncrete operations

  B. 5ormal operations

  . )reoperational

  ". /ensorimotor 

  %&planation'

  The correct answer is A. The concrete operational stage *age D+11 years- is defined by the child7s awareness

  of the conser(ation of (olume, which demonstrates that the child is able to reason in a logical way in terms of 

  the physical world. ;ote that the child does not de(elop understanding of abstract concepts until he or she has

  reached the formal operational stage *choice B-, at age 11 to adulthood.

The preoperational stage *choice -, ages # to D years, is associated with significant language de(elopment.

  :owe(er, the child has not yet de(eloped the ability to take the perspecti(e of others, and thus the child7s  thinking tends to remain egocentric.

$D

Page 48: Behavioral Science Questions

8/9/2019 Behavioral Science Questions

http://slidepdf.com/reader/full/behavioral-science-questions 48/53

  The sensorimotor stage *choice "- corresponds to ages 0 to # years and is characteried by the infant

  de(eloping increasingly sophisticated sensorimotor skills and beha(ior patterns.

A <+year+old patient is gi(en a battery of neuropsychological tests. :e scores !< on the 3echsler Adult

  8ntelligence /cale *3A8/- >erbal 8, 1< on the )erformance 8 test, and 1#< on the 3echsler emory /cale  test. 3hich of the following is the most likely site of his brain dysfunction4

  A. Bilateral frontal lobes

  B. Bilateral hippocampal gyri

. Bilateral occipital lobes

  ". Ceft hemisphere

  %. =ight hemisphere

  %&planation'

  The correct answer is ". The pattern presented suggests this person is ha(ing difficulties with (erbal material

  *3A8/ (erbal 8 of !<- but not with (isual+spatial tasks *performance 8- or memory *3echsler memory scale-.

  /ince the left hemisphere is dominant for speech and (erbal material in the maority of indi(iduals, the lesion is

  most likely in the left hemisphere.

hoice A is incorrect since the frontal lobes control socially appropriate beha(ior, se9uencing, and future

  planning. There is no indication that these are deficient in this person.

/ince memory is intact, a lesion in the bilateral hippocampal gyri *choice B- is unlikely.

Bilateral occipital lesions *choice - would produce problems with (isual recognition, which are not apparent in

  this person.

  The right hemisphere *choice %- is related to control of (isual+spatial functions *e.g., map reading, locating

  oneself in space, etc.-, rather than (erbal ability.

A $+year+old woman complains of early morning awakenings and loss of interest in e(eryday acti(ities. /he is

  diagnosed with maor depressi(e disorder and gi(en fluo&etine, but does not impro(e. Tricyclic antidepressants  and AO inhibitors are subse9uently tried without effect, and electrocon(ulsi(e therapy *%T- is

  recommended.3hich of the following represents the most serious side effect of %T4

  A. %&trapyramidal symptoms

  B. :earing loss

  . ania

  ". =etrograde amnesia

$!

Page 49: Behavioral Science Questions

8/9/2019 Behavioral Science Questions

http://slidepdf.com/reader/full/behavioral-science-questions 49/53

  %. =habdomyolysis

  %&planation'

  The correct answer is ". Although electrocon(ulsi(e therapy *%T- is highly efficacious in treating maor 

  depressions that are refractory to tricyclic antidepressants and selecti(e serotonin reuptake inhibitors, it

  produces retrograde amnesia as its maor side effect.

%&trapyramidal symptoms *choice A- are commonly produced by acute administration of antipsychotic drugs,

  such as phenothiaines or butyrophenones, not %T.

:earing loss *choice B- is not a common side effect of %T.

ania *choice - is not a recognied side effect of %T.

=habdomyolysis *choice %- does not generally occur with %T when it is performed correctly, with the

  administration of skeletal muscle rela&ants.

There is a classic oo story about a cage with three monkeys in it. The largest monkey steals the middle+sied

  monkey7s banana. The middle+sied monkey then screams with rage, hits the smallest monkey on the head, and

  then steals his banana. The middle+sied monkey is using which of the following mechanisms of defense4

  A. "isplacement

  B. )roection

  . =eaction formation

  ". =egression

  %. =epression

  %&planation'

  The correct answer is A. This is an e&ample of displacement. 8n this defense mechanism, there is a transfer of   emotion from a person, obect, or situation with which it is appropriately associated to another that causes less

  distress. "isplacement is common and often destructi(e to other indi(iduals, such as when a man is fired from

  his ob and subse9uently beats his wife or children. 8n the medical setting, the hospital staff is a fre9uent target

  of displacement when family members react to their own feelings of guilt about someone7s death.

  )roection *choice B- occurs when someone attributes his or her own thoughts to a different person.

  =eaction formation *choice - is the unconscious adoption of beha(ior opposite to one7s true feelings.

  =egression *choice "- is the adoption of beha(ior more appropriate to a younger age.

  =epression *choice %- is the deeply subconscious blocking of memories or emotions.

A #+year+old single man seeks psychiatric treatment to help him deal with difficulties in his personal life.

$

Page 50: Behavioral Science Questions

8/9/2019 Behavioral Science Questions

http://slidepdf.com/reader/full/behavioral-science-questions 50/53

  Although the man is a successful computer programmer, he feels unsatisfied with his interpersonal relationships.

  :e reports being attracted to se(eral of his female coworkers, but is too shy to talk to them about anything

  other than superficial subects such as the weather. :e would like to ask one of the women out on a date, but is

  afraid of being reected. 3hich of the following diagnoses is most appropriate4

  A. A(oidant personality disorder 

  B. Borderline personality disorder 

  . "ependent personality disorder 

  ". ;arcissistic personality disorder 

  %. /chiotypal personality disorder 

  %&planation'

  The correct answer is A. The man described is probably suffering from a(oidant personality disorder,

  characteried by feelings of inade9uacy and e&treme sensiti(ity to criticism, leading to social inhibition and  withdrawal. These indi(iduals often a(oid interpersonal relationships entirely rather than subect themsel(es to

  the potential risk of criticism or reection, although they may yearn for a more satisfying personal life.

  Borderline personality disorder *choice B- is characteried by unstable interpersonal relationships, instability of 

  affect, impulsi(ity, feelings of emptiness or anger and, in some cases, paranoid or dissociati(e symptoms.

  "ependent personality disorder *choice - is characteried by the need for constant support and reassurance,  with unrealistic an&ieties o(er being forced to fend for oneself.

  ;arcissistic personality disorder *choice "- is characteried by e&cessi(e grandiosity and an e&aggerated

  sense of self+importance, accompanied by a feeling of entitlement and a need for attention or admiration.

  /chiotypal personality disorder *choice %- is characteried by eccentricities of beha(ior, odd beliefs or magical  thinking, and difficulties with social and interpersonal relationships.

A $D+year+old, unmarried woman appears at the physician7s office complaining of diiness and pain in her 

  stomach. Although she describes her symptoms in a flamboyant, dramatic manner, physical e&amination is  unremarkable. Throughout the e&amination she is flirtatious and comments on the close and intimate relationship

  she hopes to ha(e with the physician. 3ith tears in her eyes she asks if she will be all right, and then laughs

  when the physician seeks to reassure her. :er history is significant for pre(ious treatment for alcoholism. /he

  has changed physicians four times in the past year. Based on this initial encounter, the patient7s beha(ior is most  consistent with which of the following diagnoses4

  A. A(oidant personality disorder

B. Borderline personality disorder 

  . "ependent personality disorder 

  ". :istrionic personality disorder 

  %. ;arcissistic personality disorder 

<0

Page 51: Behavioral Science Questions

8/9/2019 Behavioral Science Questions

http://slidepdf.com/reader/full/behavioral-science-questions 51/53

  5. /chioid personality disorder 

  ?. /chiotypal personality disorder 

  %&planation'

  The correct answer is ". The flamboyant manner, flirtatious beha(ior, shifting emotions, assumed intimacy, and

  general theatrical beha(ior is consistent with a diagnosis of histrionic personality disorder. ;ote that all

  personality disorders are ego+syntonic. That is, personality disorders generally do not bother the patient,

  although they fre9uently bother people around the patient. This patient may be difficult to deal with, but will  ne(er acknowledge how she constantly disrupts the normal physician+patient relationship.

A person with a(oidant personality disorder *choice A- is (ery shy and sensiti(e to reection. Although they are

  socially isolated, they long for human contact with others.

  A person with borderline personality disorder *choice B- is characteried by a (ery unstable affect, beha(ior,

  and self+image. 8nstability is manifested by splitting *seeing things or people as either all good or all bad- and

  impulsi(e beha(iors such as promiscuity, gambling, or o(ereating. 8nterpersonal relationships are intense but

  unstable. /elf+mutilation and mood disorders are common se9uelae.

)ersons with dependent personality disorder *choice - seek to ha(e others assume responsibility for their 

  li(es. They a(oid making any decisions for themsel(es and ha(e a difficult time e&pressing disagreement.

  A person with narcissistic personality disorder *choice %- is filled with a grandiose sense of his or her own

  importance. They present themsel(es as grand and infallible. 5aced with a reality that is less than grand, they

  either ignore it, or attempt to e&plain why it is really wonderful.

A person with schioid personality disorder *choice 5- is isolated and alone and likes it like that. They do not

  cra(e or seek human relationships or companionship. They may function well in isolated settings, but ha(e

  great difficulty in e(en the most basic social encounters.

)eople with schiotypal personality disorder *choice ?- are strange and idiosyncratic in their approach to the  world. Their clothing is often mismatched and their beha(ior considered odd by those they meet. )eople tend to

  a(oid them and see them as highly eccentric.

A $!+year+old actor is admitted to a maor medical center with complaints of malaise and cough. :e is diagnosed  with influena by the emergency room physician, and sent home, but demands to be seen by the hief of /taff 

  because he feels he is not getting care appropriate to his own importance. On the way down the hall, the man

  tells the orderly that he should ha(e been nominated for an Academy award, but was passed o(er because of 

  ealousy among his co+workers. 3hich of the following diagnoses best describes this man7s beha(ior4

  A. A(oidant personality disorder 

  B. Borderline personality disorder 

  . "ependent personality disorder 

  ". ;arcissistic personality disorder 

  %. /chiotypal personality disorder 

<1

Page 52: Behavioral Science Questions

8/9/2019 Behavioral Science Questions

http://slidepdf.com/reader/full/behavioral-science-questions 52/53

  %&planation'

  The correct answer is ". The man described is e&hibiting the signs of narcissistic personality disorder. This  condition is characteried by e&cessi(e grandiosity and an e&aggerated sense of self+importance accompanied

  by a feeling of entitlement and a need for attention or admiration.

A(oidant personality disorder *choice A- is characteried by feelings of inade9uacy and e&treme sensiti(ity to  criticism leading to social inhibition and withdrawal.

Borderline personality disorder *choice B- is characteried by unstable interpersonal relationships, instability of 

  affect, impulsi(ity, feelings of emptiness or anger and, in some cases, paranoid or dissociati(e symptoms.

"ependent personality disorder *choice - is characteried by the need for constant support and reassurance

  with unrealistic an&ieties o(er being forced to fend for oneself.

/chiotypal personality disorder *choice %- is characteried by eccentricities of beha(ior, odd beliefs or magical

  thinking, and difficulties with social and interpersonal relationships.

The parents of a D+year+old boy di(orce. The boy li(es with the mother and sees his father e(ery other weekend.

  "uring these (isits, the boy is alternately sullen and angry with the father, but when it is time to return home, he

  clings to the father and cries in a desperate manner while saying 87m sorryP 8 want you and mom to li(e together   again. 3hich of the following is the most helpful statement that the father can make to the son4

  A. Big boys don7t cry.

  B. 8 left your mother, 8 didn7t lea(e you.

  . 87ll see you in two weeks.

  ". Nou7re the man of the house now.

  %. Nour mother was too hard to li(e with.

  %&planation'

  The correct answer is B. This statement from the father would reflect his understanding of the egocentric

  nature of school+age children. That is, the child is assuming that he is responsible for the di(orce between his

  parents. The anger and withdrawal reflect the child7s frustration with the situation, but the tears and apology

  suggest the child7s fear and assumed responsibility for the breakup.

Big boys don7t cry *choice A- is a demeaning and belittling statement.

  87ll see you in two weeks*choice - ignores the child7s felt responsibility for the di(orce.

Nou7re the man of the house now *choice "- places too much responsibility on a D+year+old child.

  Nour mother was too hard to li(e with *choice %- places all the blame and responsibility for the di(orce on the  parent with whom the boy li(es on a daily basis. 8t ignores the reality that di(orce is usually due to difficulties

  that both parents ha(e with each other.

<#

Page 53: Behavioral Science Questions

8/9/2019 Behavioral Science Questions

http://slidepdf.com/reader/full/behavioral-science-questions 53/53

<